lain A M Hennessey
MBCha (Ho,n)'" Senior House Officer, Ne()nol
Alan G Ja pp
MIlCh. iHoNl SS< IHomI MlCP
Clinical Research Fellow in Cordiology University 01 Edinburgh, UK
bLSEVlFR
IH='I M'eS
CHURCHILL LNTNGSTONE CLSEVlER
Cl2OO'l, Dsorvitr l.I.n,tro. AU rip'" ~
No part of th'''I'"bhcation may u.. ..."rodll<.Wt. !;I,,,....j in ~ rWicval ~)'I~, 0< tnnmUnl~11tlany form or try .ny meam, el".:tn...ue, mf!<:hanica1.I'OOtooop)'i1le;.. t«Oidi"g or oct..... ..-be.... ,ltnut tht prior ' .....1IlUIIioon ci ttr PubbIhen Pen",,,,,,,,,,,, may be .>Ught directly hu'" FJSorrier" s Hrolth Scie>c6 Right>. U'1,aliment, 1600 luhn F. J(ennedy RlIUI""'ard. Suit(' IHOO,Philadelphia, FA 191W-2IW9. USft.: phc:lne: (+1) 21 <; 2.19 3804; law' ( • I) 215 2:)9 3lI05; or, ...m.1.il; IIIlIII1tJtpa.e k_~ IN'\' abo C'
-broe ' 101 !he ~ homrpIo~ {h"P'1 ! _ ...·,~-.'""l,b,- oIekctuq; 'Support and CQnt
, '"
fSDN·13, 978-().-41::1-1{).l14-S (M,,,,, Echboo) 1SllN-13, 9'lS-Il-«3-1041311 (lnn-'.....l>l)na/ E.dih'"') Brili$h 1.......uyCatAl..S'''''1l. in ..... bllc"ho.. Oau Acnto'lngue r«ord fc.w tillS book is ....a,Loble from the
KTlt~<Jl
Libruy
l.ibrwy 01 Consn- e.taJosb's in f>ublicatiM D..... AQtalos n-o:
"""'"
N("Ott- the publisher ..... 1ho! ltuthon. _l.U'ne' any ~ ....bility 1m any Ie,.. or inJurY ./CK ~ 10 ~ or i"'OjA' Ii ari$mS' "'I 01 or n'Latl:'d 10 M<' use of the mal~1 coot.>ined If1 thos book. It is thr.o n:sp<JnlIibility of t1.. laallng prKti~ "'lyin,l; on indcpl~ldenl~''''Kl knowl~ge of U'" pat~t. to OtlaDWv-lhe mllreatrn<.. ,1 and method of applKatlOll foe tht",,"IHnL 7\rPtoW......
WO,.king together to grow libraries in developing countries --.doc¥icr.cocn
I
WW'W
boob.,J.~
I __OAbK org
..
your soun:. fw boolu. jo.-na& and ndtio,..... ~L:lE"IFR In tt.. tMotth sclenc:es
_etse\fierheoltb.com
ll'l'? ..
P*Y_"_ ,.,., • •
.....
I I
r
I
"II:
I
Preface If you've taken the time to open Arfnwl Blood Gases Mnd, l-:tJsy, you must believe that artenal blood gases (AHtJi) are unportant. but not Pntirely:,lraightforward. We ft>rtamly agn..~ on the first pomt: ABC analysis now play:.
il/1
indispensablf' role in the assessment and manilKcmcnt of pahf'nh; wltll a huge range of acute moollal and surgu"al problems. AccuratE' ABG interpretation is undoubtedly a fundamental skill U1 modem clink'" medianl::. On the SI.'('ond pomt, we hope this book ran bto of a~sislance. Throughout. our aim~ have ~,\ to emphasi<:l' the key concept~, fnrus on priKtical and useful aspects of ABG analysi!! and dvoid extrant"Ous detal1. \Vc believe lThlny medical and nursing students, JWlior doctors and specmhst nurses will benefit from a dear, roncise guide to performing thl? te<:hnl4uc and interpN>tmg the results lain A M HeJUlCSSCY Alan Glapp
•
Contents Prefcx:e
v
Acknowledgements
V"
1
Port 1 The ABG explained 1.1
3
Introduction
1.2 Pulmonary gas excnange: the basics 1.3
Disorders of gas exchange
1.4 1.5 16 1.7 1.8 19
Acid-bose balance: the basics Disorders of ocid--bo$e balance
ABG $Ompliog technique
When and why IS Common values
on
ABG required?
Making ABG inlefprctolion easy
Appendix Pori 2 The ABG in practice
•
4 18 26 36 42 48 50 52 56 57
Answers
58 109
l<>de.
135
Cases 1-25
1. 1
..
INTRODUCTION Arterial blOOl"l ~,\S (ABC) analysis ~fers 10 the mcasuremE'nt of pi I and the partial pn:s~.m:s of oxygen (Oz) and carbon dioxide (Co,) in artt>rial blood. From these values we can ass....s ttwo state of tJdJ--buse bolanCt in blood and how well lungs are ~riorming their job of gll5 ,:xc}Ul11gt.
Already the~ afC questions: what is meant by 'acid-base status'? What is.:l 'partial pressure'? Why do they matter? It helps to break things cJown.
Part 1 of this book is designed to answer the:;e questions. We start with a few p
3
PULMONARY GAS EXCHANGE: THE BASICS OUf ~lls use oxygpn (0,) to generate CJ\cq:;y and produce carbon dioxide (COz) as waste. Blood <;upphes cdb with the O 2thfoy need and dears the unwanted COt. This p~<; d~ds on the ability of our lungs to f'nnch blood with O:z and rid it of Co, Pulmonary Ras rxcllatlg' ",fel'S to the ttdusfer of O:z from the atmosphe~ t~) the bloodstream (oxygenation) and CO 2 born thp bloodstrp-am to the aunosphcl'c (C02 e-limination).
TIle cxchan~e lakl"S plal'"P between uny air sacs call1''d alvroll and blood. \'t$scls called mpilfl1n~. Because ~y carll haxe exlTf'mely thin walls and rome into very d06E' conmct (the al\'coIar-capillary membrane), Co, and O:z arc able to move (diffu~) betw~ them (Figure 1).
-'" bI ollctliole
I
02 and co, lranSlBr occurs at the aM!dar A F~ure 1 Ra.pimloty (Jnolomy.
•
B
capIafy l,lfl!OOrar,I
PULMONARY GAS EXCHANGE: THE BASICS
ABC.s help us to assess the ~ltectivencssof gas exchange by providing measurernt"tlts of the partial pmiSurn of O 2 and COz in arterial blood - thE" PflOz. and Pa~. Partial pressure dcscribe5 thE" contribution of on~ individual gas within a gas IJ\i.Jl.ture (such as air) to the total pre;sure. When a gas dissol\l~ m liquid (e.g. blood), the amount dissolved depcnili; on the partial pt'PllSUTe.
Nole ~.
partial pressure of 0 1 PalJl '" partial pre$5ure of O 2 in
D.rl~ri.l
blood
Gases move from areas of higher partial pressure to lowE"I' partial pICSSW\!. At the alveolar-
A~on··· . . .
AI
:leO
P_uU""
1eYeI, otmospherk. pro!oSUfe (klkJl p"euure of gases
alrno$pn..1 - 10 1 lPo
Of
in
the
760 mmHg
o? oompriSC5 21 % of air, so the porliol pre$sure of ~ In oir • 21% of otmospheric. pnmure
.21 kPoor l60mmHg Co, mcJI,;~ up in~,ed
aIr IS
fV1l a
til"/)'
frodioil 01 air, so the pamol pressure of C~ in
negligible
•
f'Ut.MONARY GAS eXCHANGE; THE 8ASICS
CARlON DIOXIDE WM''''ATION Diffusion of co, from lhe bluodstrPam to alveoli is so dficient that COl elimination is actually limited by how quickly we can "blowoff" the CO2 10 our alveoli. Thus, the PilC02 (which reflE'Cts the overall amount of C~ in arterial blood) is dttermined by al'Ot.'Olar Drnhlahon - the total volume of air transported bctwt.:cn alveoli and the outside world every minute. Vmtilation IA regulated by an area m the bralnstf>m called the respiratory centre. 'This area containS specialised rca::ptun. that sense the PtlCOz and cvnnect with the muscles involved in breathing. If it is abnormal, the respiratory centre adju~ts the rate and depth of breathing acrordingIy (Figure 2).
Normally, hmgs can maintAIn a normal P~ even in rondltioru; where Co, production i!'l unusually high (e.~. sepsIS). Consequently an increased PIKOz (hypercapnia) always lD1plies reduced alveolar ventilation.
Key point
PaCo1 is controlled by vtnlil.llion ~nd the level of v..ntilation is ildju5ted to m.aintilin PaCo1 within tight Iimils..
•
I •
PUlMONARY GAS EXCHANGE: THE BASICS ~Paco2" +V&nlilatioo
t PaC02 " tVentla\loll
Figure 2 Control of ventilation.
~ note on ••• hypoxic
drive
In patients with chronicolfy high PoC02 levels (chronic hypercapnia), the ~iali-ed reo:;eptors thot deled CO 2 kmlls can become de$ensilbed. The body then relies on receptors that detect the POiJ-l to gouge the adequocy of ventilation and low POOJ becomes the principal Vflnlilotory stimulus. This is referred to as hypoxic drive. In patients who rely on h~ic drive, overzealous cOfrection of hypoxoemio, with supplemental O:z, may depress ventilation, leading to a colastrophlc riM in POCO:!. Patients with chronic hypercapnia must therefore ~ given wpplementol O 2 in a controlled fashion with careful ABG monitoring. The same does not oppfy to potienb with ocut. hypercapnia. 7
PULMONARY GAS EXCHANGE: THE BASICS HAEMOG~O.'N
OXYGEN SAtURATION (SO,)
Oxygenation is marc complicated than C~ t'liminabon. 1lte first thing to realiSE' lS that the Po:!. does not actually teU llS how much ~ is in blood. It only fTlf'asures free, unbound ~ moIecules- a tiny proportion of !he total In fact, almost all 0,. molecules in blood are bound to a protem called hamwgfobin Hb (Pigure 3). Becau~ of !.his, the amount of O 2 In blood depends on two facton.: 1. Hb concentration: this determines how much O 2 blood. has the capaCIty to carry. 2. Saturation of lib with 02 ($0,): this is the pcrccntaj.;c of available bindin~ sites on lib that ront3m an 02 molecule - i.e. how much of the carrying capacity is btrng used (Figure 4).
Note
Soz:: O 2 wtueation in (;my) blood $Q:O:!. ""
O 2 "tueation in Q:rlmal blood
It. note on ... pulM ~i t
So::>:2 con be
me<»ured using
0
Pfobe
(pul!oe o..imelerl opplied /Q the
finger or earlobe. In most case$ it Pfovide$ odeqUQte informatioo Q,ll,ygenarion,
but it
10 901.19-
iller.s occurate witn saturatlon1 below 75% and
00e$ not provide infoflTlOlion on f'acQ) w ~hwklllQr be u~ 0$ 0 wbditute for A8G onolyJis
unre!iQble when plIfipherat perf",lion il poot. Oximetry
In
ventitololy impoir",..l.
Key point
.Po,. is not a m('uu~ of the unount of O 2 in bJuod - ultimately the SaO:!;;md Hb concentration determine the D,. confent of arterial blood.
•
PUlMONARY GAS EXCHANGE: THE BASICS
O:!tnnlklHb(M)
Ft,ure 3 Relative fXoporliom of Ire. 0, moIecot.s and O 2 moIecuUu bo<md b hoomoglobin in blood. HaemogIobil 1TlOIea.M
Fre9 QIl)9lll rnoIecUe
Boln:I axygen ~
•
,
,
•
,!,
•
i
,! ,,
•
"
!
,
• I,
•
I
• •
I ,,
• •
11::'=1 Figu... 4 Haemoglobm
11~~=1
I I
I~=I
OX)IQ~ sol!Jfation.
•
PUlMONARY GAS EXCHANGE: THE BASICS
OXYHAEMOGLOBIN DISSOCIATION CURVI Vk now know that the amount of ~ in blood depenWi on the Hb COTKt.>nlcation and the~. So what is the significance of the PQ:l?
Po;. can be thought of IlS the driving force for O! molC'Cull.'$ to bind to Hb: as such it regulates the Soz. The oxyhapmoglobin dissociation CUC'Ye (Figure 5) shows the So,. thai will result from any giV01 Po,.. In genern,
higher the Paz, the highel" the Sov but Iht CUrtlt is not littttlr. Tfw. green line is known as the 'flat part of the cwve': changes in ~ over this rangf' have relatively little effect on the 50:1. In contrast, the red line is known as the 'steep part of thf' curve': even small c.hanges in PO]. over this rangf" may have a major impact on~. ~
Note that, with a 'normal' PIlO:l of around 13 kPa (100 mmHg), Hb is, more oc less, nUlximally stlhmdm (~> 95%). This means blOl.X1 has uwd up its D,-t:acryinK capacity and any further rise in Pao, willlWt significantly increase arterial D.z content
Key point
Po" is not the amount of O 2 in blood but is the driving force for 5.lItunting Hb with 02"
I.
PUlMONARY GAS EXCHANGE: THE BASICS 100
Cww shifts to IIfl wtleft:
!pH.IPro, *lemperaIln,
80 RJDPG
Curve sIllfta k) righI wMri:
IpH.lPoCO" i'twnper.lIllI'e, 12,3OPG
" o+-,-~-=~_~_~--,_,~ o 5 10 15 20 80 ~~
,
o
37.5
7S
112.5
150
600 ~rrmtg
Figuro S ~Jobjn disso:iotioIl anotl. The CUfVe defines the ,J.m.,,,sJ.ip beMun Po, and the percfflJloGe so1uI"ation 01 haemoglobm WIth 0")'9$l (~_ NoIe the sigmoId sho~: it is relolive/y Rot when P01 's >
80 mmHg flO 6lPa) but sleep whoM Po, foils below 60 mmHg fS lPo)
Key point
When Hb approaches maximal O 2 saturalion, furth~r increalles in Paz uu not significantly increase blood O! cuntent.
PUlMONARY GAS EXCHANGE: THE BASICS
AlVEOUR Vllffll.ATlON AND ~ We hav*, now seen. how
P«>z re,;;uJa~ the ~. But what dcl'erlniJ'w>
P«>z' There are three major factors that dictate the Pac,;: 1, Alveolar ventilation
2. Matching of ventilation with ~rfusion (VIQ)
3. ConCUltntion ot 0 1 in inspired air (FiOa) Alveolar ••ntilation O:z moves rapidly from alveoli to till: bloodstream - so lhe hIgher thr uluwwr Po" Ihe hixhtr the PIlOz·
Unlikp air in the atmosphere, alveolar air rontain.'l significant amounts of CO:z (Figure 6). More CO:! means a lower p~ (remember the partial pressure of a ga" reflects Its share of the total \'olume). An mcrease in alvoo1acventilation allows more CO:z to be 'blown oH',
resulting in a ~ alVl."'Olar ~. If, on the other hand, vcntilatiorl. declines. L~ accumulate; at the cxpenw of O:z and alveolar ~ falls. Whereas hyperventiation can mcrea!le alveolar pO:! only slightly (bringing it closer to the Po:z of inspin>d air), there is no limit to how tar alveolar f'oz (and hence P~ can fall with inadequate v.:ntilation.
12
PULMONARY GAS EXCHANGE: THE BASteS
"" 75"
NftgIl.
"'-'
"""""....... .....
-
75"
.....
,tgur. 6 Composrlfon 01 inhaled ond ..."haled 905& at voriou$
~$
oJ
respiration.
K.y point Both uxygen.ation .and COl elimination depend on alveol.u ventil.ation: impail'e'd vcntiJiiltion cau5e5 PliO, to fall and Plica, tu
ri,..
13
PUlMONARY GAS EXCHANGE: THE BASK:S
Ventilation/perfusion mismott:h and .fhunring Not all blood flowing thmugh the lung meets wt>U-venhlat~ alveoli :md not all vt:ntilatcd alveoli ilrt' pt'rfused with blood - E"Speeially Ul tht> prest>ncp of lung disc~. This problem is known as ventilation I perfusion (V /Q) mi'lmatch. lmagme if alveoli in one area of lung are puorly venbJittPd (e.g. duc to collapse or consolidation). Blood pitssmg thot' alveoli n>tum.~ to the arterial circulation With Ie::;:; 0, and more ('01 than normal This is knuwn as shunting l Now, hy hyperventilnting. we can shift more air in and out of our remaining 'good alveoli'. nUs allows them to blow-uff eXlra C~ SO that the blood passing ttwm can offload more CU:!. 'The luwer Co, in non-shuntrtl blood compertSittH fOf" the higher COl in ..hunted blood, m;untaming tht! poleD,. The :,ame docs NOT apply to uxygt!J\i1lion. Blood passing 'good alveoli' is not able to carry more 0, Mause ils hM>moglobin i::. alrt>itdy maximally :xIturatcd \,nth <-1:1 (remember: flat part of curvt:, page 11). Thf' non·shunt~ blood lhenofoN' carumt rompensah> for the low O:z levels in shuntt"d blood. and the p~ falle;.
Key point V/Q mismatch allows poorly ol()'genated blood tu re-cnter the arteriill circulation, thull low~ring Pao" and Sao2" Provid~ ov~rall
does not
I~ad
il)veolar ventilation is main~ed, V/Q mismatdt to an inC~ilse in PacOr
'Tl\(> tPTITI also applies to bluud lh"t bypasses alveoli altogelher (anatomICal ~'W1IUlg)
14
PULMONARY GAS EXCHANGE: THE BASIC",S,--_
.J=~=I ''''''''' O2
so, 11m
so,
",,.
..CO2
'"
'''"'
.....- -
--
so,
so,
....... CO2
15
PUlMONARY GAS ~CHANGE. ,~
mr BASICS
and oxygenation
The fraction of inspired oxygen (FiO:J refers to the percentage of 0: m the air we breathe In. The FiOz in room air IS 21 %, bul can increast"d wIth supplementaJ~.
"*
Aluw PacJ.z may result from t'ithcr VIQ mismatl..:h or inadequate vrnblabon and, in both ca~, increusi.njl, the F~ will improve the PaOz. 'l~ ~)(act r~ rcquirem..nt vane!:> depending nn how severely oxygenation is impalrM and will help to d ~ the chokP of deli...,cry device (figure 8). When thf' cause is inadequate ventilation it must be remE>mbered that incrc3Sing riO, wIll not rev~ the rise in P~. Supplemental ~ makes ABC analysis more complex as it can be difficult to Judgc whether the P/Kh is appropriately high for th~ F~ and, hE"llet", whl.>thcr oxygt"nabon lS impaired. A useful rule ot thumb is that thp diffeft'J)(.'C betwffn r~ and POOl (in kPa) should not normally be ~r~ater than 10. Howevpr there is often a d€'gree of uncertamty as to !he pl"€'Cise FiO,. and, if subtle impaU"lucol is suspected, thf' ABC should be ~..ated on room air.
OXYgMI et.livery
devic.,
Nasol fNO"gt. f~ < .. ~. ComfOl'tol:h ond comenient FiO, specific: depends on flaw rem t1-6 Vmin) and venhlabon
n0n-
Stondard fm;" mask: fiO, 30-50% at flow rotel6-10 Vmln bul ImpuKise Mot COllie CO:! relention ot flow, < 5 Vmin ('rebreolhing'). not 1IJefu1 Jar providing 10- ft0:2
100
~ rno~ F~
24--00%. Ddivers fUced, prediclabl. Fi0 2• Ideol for providing controlled, occurote O:z therapy allow CorN;ef\lfQl;QI1S fixH p«fOiilNilKe (high&wl
meuk wiIh re••nooi. ft0:2 6IJ...8O".li.. Con ochie¥e _ higher fiO:z with IighHifhng rnosJc Useful for ano.llefm UM in respiratory foe.
emergencIes fndotracheGl intubation: fi0 2 21-100%. Uled In soyerely un......11 potierlb with very high ~ requiremenls, especioMy wifh venhlolory !oillire. Patient is sedoltid ond InlI<::hookolly ventiloted.
,.
PULMONAAY GAS EXCHANGE: THE BA5K.:S
Fbled pelb Ifs ... milt:
VaoobIc perforrnarx:e mask
Figun a Oxygen deJ,WKY clevius,
17
DISORDERS OF GAS EXCHANGE HYPOXIA, HYPOXAEMIA AND IMPAIRED OXYGENATJON The above lerms aN': often used interchangeably but mean different thmgs. Hypoxia refer.; tu any stld~;11 whIch tissUt'S M:'riot lI1I iruzdeqlUlk supply of 0 1 to support nOrmQIlJt7'Obic mrl/lbolism l (Figwe 9). It may result
from either hypoxl!emia (!lee below) or impaired blood supply to tis~ues (isl.:hacmia). It is ('Iften associated with lactic acidosis as cells resort to anaerobic metabolism.
tmy slQf~ in wIridr 1M O 2rontent ofQrlmal blood is reduced. II may remit hom impaired oxygenation (see below), low haemoglobin (anaemia) Of reducw affutity of ha~mog1obin for Oz (e_g. carbon monoxide). Hypoxa~mla refers
to
Imp.tired oxygr:nation refers to hyporMmJII fC>.41tingJrom rtduaJ. tnlmfer of D1from lungs to the bloodstrtflm. It is identified by a low P/Kl:!, « 10.7 kPa; < 80 rnmHg). It is important 10 note Ihc distinction betWt:'ell impaired oxygenation (which results in hypoxaemia) and IrwdLqllllteoxygcnation (whkh re;ults in hypoxia). Constder a palimt with a PlIU:!. of 85 kPa. He M..'l impaIred oxygenation. suggesting the prcsmce of irnportanllung diSE'OlSf'. NeveI"lhtllt.-ss, his Pao.z would usually result in an ~ > 90% and, protlidt'd th£ haemoglobin and (llrdiac output are normQl, adequate 0:l: delivery to tiS$Uell.
lit" often m.lvisabl(' to u.'\t! the term 'm.."uc hypoxia' to avoid any l't>nfusion.
11
rnsoRDERS OF GAS EXCHANGt:
5pee*-(e.g. "'JOC*cIIl ila.......,)
.,
.<
.,
iJ
t_I__~_ _-t HoomogIobil
low /1QernagIObi'I
low ~
I.WlIlbIe 10 carry ~
(~)
(1IIp"oed arygemIion)
(e.g. CO poisoIwlg)
figuN 9 Cau.seJ 01 hypo;ua
••
DI~DERS
OF GAS EXCHANGE
TYPE 1 RESPIRATORY IMPAIRMENT Typt! 1 respiratory impairment' is defined as low P~ with norum! or low P~. This implies defective oxygelation despite adequate ventilation. V,Q mismatch is usually responsibl4" and may tc5ult from ;1 numbt"r of causes (Box 1.3.1). The PIIJCO.z is often low due to compt!J1satory hyperventilation. If the ABG is drawn from a patient on supplM'lf>tltal Oz. the Pl7O-z may not be below the normal rangt", but will be inappropnately low for the FiO,.
The Sf>verity of type 1 respiratory impainnent i~ judgt..'l:1 according to the scale of ~ resulting hypoxaemiol and, ultimately, the presence of hypoxia (fable 13.1). Hpre it IS important to rernmilicr the 0,. dissociation curve. Reductions in PtIOJ as far as 8 kJ'a have a relatively minor effect on Sao:z and are well tolerat~. Beyond this threshold, we reach th£' 'steep part' of lh£' curv£' and further reductions in PtIOJ will lead 10 much greater falls in ~ significantly lowt'ring the ~ content of arterial blood. Initial treabm.. . .l l of type' 1 resplJatvry impairment is a~ at achieVing an adequate Pac;. and Sat~ Witll supplemf'ntal O 2 while atlcmpting to COIn:d Ute W\derlying cause. In many ca5f'1l pube Olomt'try can be used as an alternativE' 10 repeated Aac; sarnplmg to monItor progress.
'We USl' thf. lftm 'inlpairu-."ll' ralMr than 'fai.lure' hL-rc as t~ diaSJ'OSlS 01 re5pU"ilkw)' failUT# requires I Pao:z < 8 kPa « 60 rnmIiX).
DISORDERS Of GAS EXCHANGE
So;.:
1.3.1 Common CQU5e'5 of~ 1 respUoIwy lmpai.....ent· ~ Acula osllullo Acvte re$piratory disben syndrome Flbrosing ohieolitis Chronic OO'l'U<:llve pulmonary diseose
Pneumonio Pulmonary emboli$ll\
........ho<= Pulmonory oedema
Table 1.3.1 Auesdng MYerity of type 1 respiratory impai,""*"t
POOz {kPol
POO2lmmHgj Sao, 1%1
Mild
~Clte
Sovwe
8-10.6 6f>-79
53-79 40-59 7>-89
<53
~94
at". _ken of .....e... intpo........t • • •
<40 <75
.
H'9h f~ reqoJiremenl$ to maintain odeqvole Paa, loctic: ocidl)$i$ lindicOling tissue hypolliol Orgot'! dysfunction (drOWSlnen. confulion, reool fo,luf., hoemodynamlC coIIopMl. como]
21
DISORDERS Of GAS EXCHANGE
TYPE 2 RUPIRATORYIMPAIRMENT TYJX' 2 respiroltOry impairment is defined by a high PoX..U:l: (hypercapma) and is due to madequate alveolar ventilation. Since oxygenation alw depends on wntilation, the Paut is usually low, but may be oorffial if th.. pabenl is on 'iUpplemcnlal 07' It is important 10 notp that any cause of typt! I impairment lllay lpad to type 2 impairment if exhaustion s\lpervencs. Arnte rises in Parll:! lead to dangpmus accumulation of add in the blood (see ~'Clion] .4) and must be rf'\·er~:tJ. Chronic hypercapma l:> olccompamed by a rise in bicarbonate (HCO~), which pr~rvcs aCidbase balancp. Howcwr, p"tiPTlts with chJ'()r'lic type 2 irnpaUID\.'n1 who expcrienn> a further sharp decline in ventilation will also holYC a rapid rise in PaC02 tacute Oil chronic), le~ding to aCid .I.ccum\llation .md low blood pH (Table 1.3.2). Supplemental ()2 improves hypoxaemiJ; bUI nol hypercapma, ~ trf'atment of type 2 ft>Spuulory impainncnl should also include measures to improw \'ffitilation (e_g. w.ersal of sedation, reJjpf of airway" ~Iruclion, a';si"tetJ ventilation). Thc OVPrzealous lldTlunistration of supplemental 0,. to som!' patients with chnmic ty'pf' 2 impainnent may furthcrdep~'iventilation by abolishing hypoxic drin> (p. 7). Pulse oximetry provides no mfurmation on PIICOz so is nut a suitahle sub:.tilutC' for ABC monitoring in tyJXl: 2 ~uatory impairment.
22
OIS<:)iOERS Of GAS EXCHANGE Tobie 1.3.2 The ABG in dfff--.t pullern, of type 2 impairment
- - - -Po«>, -
A
Chronic
Acvte on chronic
HCO,
•
f f f
pH j
-
f f
I
Bolt .1.3.2 .~I1'l~ (QUMS of ~ 2 reJPiratory.lmpoirment
Chronic O&tTur;tive pufmonory di*Jse"
Opiotel'benzodiozepine Iollicily
hho",hon flail cn.d injury Kypftoscoliosis
Inhaled
BoX 1.3.3
body NeuromusctMar disorders 0bWuctn. sJeep OPI(NQ foreign
-
Clinical ~ of hypercapnia Drowsiness
Bounding pulse Heodoche
.3
DISORDERS OF GAS EXCHANGe
HYPERVENTILAnON Hypervl'!nlliation leads 10 a low Pan>..> (hypocapnia) and a corre:por.ding ri~ in bluud pH (set> Sfftion lA). In chrot"Jc cases it is accompanied by a ri'it" in HC0.J, which corrects blood pH. An increaSf" in the ratl: olnd depth of breathi.ng is usually apparent. A large drop;n Pan" may lCpiratory disc ABC J>hows a low PIILUz with a normal p~
H)'peJVenhlahon abo occurs a5 a compensatory response to m~tabolic acidosis (secondMy hYP*'rvpnhlation), as dE"lCribed in section 1.4. Other caUS('5 are :ihown in Table 1.3.3.
Table 1.3.3 Common causes of
Pllmory
Anxiety u»ydq,enkl ...".,..,...~
Solicylote lollicity
hyper~llilmion
._---
Po,n or diYre55 f.." Cenlro] I'ler¥OU3
s~1em diK>rc!e'l
HepatIC cirrhosis MetabolIC (X~
101 "'" oohologyt
I
DISORDeRS Of GAS EXCHANGE
SUMMARY Of GAS EXCHANGE ABNORMALITIES T'ht> four lllai.n putt~rns of ABC abnormality in disorders of gas exchange are summarised in Tuble 13.4
Tabte 1.3.4 Poo, IespualOl"f t' . • ..nt '"'PO*"
TYP" 1 Aculo type 2 Chronk type 2 *
lIII-
-
Hypervenlilolioo °Acvle on dlfonk;
,
di.lingui~ From dwonic
1'0<0,
1/I I 1
bv pre..,roc. 011
HCO,
-I
./1
H'
A note on ••• the A-a gradient
The A-o gradient i$1hc diK.rencll bet...ee" the alveolar fIt? love
In proclice its moin
\I$OS
2 rc$puafory impairment {thi~ distlnguiws pure Iype 2 respiratory Impairment from mi:wd type I ond type 2 Impairmentl
Cob.okJtioro or the A-a gradient i~ not required for the _1'0001 in Port 2 of lIlll book but, for !hole inlafel
ACID-BASE BALANCE: THE BASICS The terms aridity and alkalinity simply refer to the «()JlreIl.tration of free hydfoR,en ions (H') in II solution. H' concentration can be ~xp~.J wro:l1y in nanomoles per litre (nmol/L) or as pi I (sover)..
Solutions with hIgh W (low pH) are acidic and those with low H' (h.igh pH) are alkalinf'. The POint ilt which a substance changes from alkah to acid is lhc neutral point (pH :z 7, H' s 100 nnwl/ L). An i'ldd is a 'iubstance lhat ,dea5f!S H' when It l~ dissolved in ~lution Acids ttwrefotf" increase ~ H' concentration of the solutiun (lower the pH). A base i.~ a sub:.tancc lh.'\1 a«tpt.'1 H· when
dis.c;olved III solution. R"lSeS therefore lower the H' concentration of a solution (raUlt! the pH). A buIll?( is a substance that can either ;lea-pI or releilSt> H' dependillJ!!; on the surrounding H' concentration. Buffers lhcrdon> rl'Sist big changt:lll in H' concentration.
Human blood normally h.\5 a pll of 7.35-7.45 (II' _ 3..~.5 nmol/L) so is ::o1ightly alkalinE'. If blood pH is below the normal ranFje « 7.3..'), there IS an addacmia. If it is above the normal range (> 7.45), there is an alkalaemia. An acidosll'l .." any pnx:ess thallow~ blood pH whereas an alkalosIS
is any pt'ClCeS6 that
2'
r3lSeS
blood pH.
AClD-aASE BAlANCE: THE 8ASICS
What as pH? The pH (power 01 hydroget'l) Kale l~ a ~Implln.d 'W'a'f of expreuing large ~ in H' =ncerflluhoo, though if you'... not corne ouou jf before you
might think. II 'NO$ d.t.iglMd iuu to
confu~ you!
It i$ a MgUtive logarithmic KoIc fFigvfe 101. The 'negative' meon$!hut pH ~ get lower as !he H' concentration increoMli flO a pH 01 7 1 ;$ more
OCIdic thon 7.21. The 'Iogarilhmic.' meons thol a Mlift in pH by one numbel r.prelents a 10-f01d chonge in H' concenlfolron (so 7 Ii 10 times more ocidic than B)
Why i.
aci~s. bolance
important?
J;or cellular pI"0CCSS4.'S to occur effiCIently. Ihf. II' concentration lJ\USl be kcpl within tight limit!!. Failure to maintain pH baJarn.."C Ical.!s 10 lIlefficient cellular reactions and ultimately death (Figure lO).
pH
..
735 6.9
'16
7JJ
100
I.'
12
""
6J.l
7.3
1.45
I I
75
1.6
71
1~.6
25.1
20.0
7A
I
Sl
:5.8
"
35
I~
'5.8 Alu'csis
... , is
..... , . pHM """
27
ACID-BASE BAl.ANCe THE BASICS
MAINTAINING 4CID-8ASE BALANCE What generates H+ ions in our bodies?
n.e breakdown of fats and SUgMS for energy generates ~ which, when dis:dvro In blood, fOf"DlS carbonic acid (see Box on page 211). M~abolism
of pmtPin produces hydrochJonc, :lulphurit: and other so-called 'metabolic acids'
H· ions mu.st, therefore, be rcmovL'd. to maintain normal blood pi!. What removes H+ ion. from aur badies7
Respiratory mechanisms Our lungs are responsibl~ for removillK CO2_P'ICO:!, the parti..,1 pressure of carbon dioxidt> in our blood, is determinoo by alvL'Olar vCl1tllo:ation. If C~ production is altt"red, we adjust our breathing 10 exhale more or k'SS COg as IlI.'."CSSary. 10 maintain PlIC'O;z within oormallimifs. The bulk of the acid prodlK.L'd by our bOO.ies is in the form of COz, so it is our lungs that excrete the vast majority of the o:aud load.
Renal (mefobollc) mechanisms The kidneys art' r(>$poru;ible for eXcrelmg rnmbolic acids. TIleY ~'l,;rctc H' ions inio urint> and reabsorb HC0.'l from urine. HCo, io:t a base (and therduA:' <1<.:CCPlS H' ions), so it reduct'S the ("onr('ntration of H· ions in blood. The kidn~ys can adjust urinary H' and IlCo, excretion in responSE' to changes in metabolic led pnxluL1ion.
ACID-BASE
BAl..ANC~:
THE BASICS
MAINTAINING ACID-BASE BALANCE The l'Cnal and respiratory syst..ms operat", Jomtly to maintain blood pI-{ within normallirruts. If one system IS ovtrWhelmcd, leading: to a ch;mge in blood pi I, the otl'llo'[ usually ddjUSl$, automatically, to limillhe dlSturh;mce (e.g. If kidneys fail to ~cretp m~abolk acids, ventilation is UKTlo'a..'led to exhale mure C00. This is known all wlilf!t-1ISQUm/ Importantly, lvrnpensatOry change1l in respiration haprt"" over minules to hours, wherc mt>taoolic rcspou!>eS tak(> days to develop.
Jwt one equation•••
~--This one eqoolion is c:rucial to understanding ocid-ba.se bolonc:.; ~o +
CO2 +-+ ""Cal .... H' + He0s
Firs/ly It show51hot C~, when dissolved In blood, becon'lfts on
The mOle COz added 10 blood, the more H1CO, lcorbonic oc,dl is produced, wIloch dissociates to release free W ions S8ct:Jldy, it Pfedicts Ihot blood pH depends not on the obsolut.t ornoonts or Co,; 01 HCOl present but on 1M 'Otio of CO2 10 HCo.,- Thus, (] change in C~ will not Ieod to a c:horoge in pH if III, boIonced by 0 c:honge in HC~!hot p'esetvei Ihe ratio (and vice ....nal· Since C~ is c:ontrolled by respirolion ond HeO, by renol~, !his expkJlnl how cornpensotoon con pr~t
c:honges in blood pH.
29
AClD--BASE BALANCE: THE BASICS
DISrURBANCES OF ACID-BASE
BA~LANCE""
An acidOl'iis is any process that acts to lower blood pJ I. II it is due to ;; lise in P~0z.it is calied a r£~pirator.LJ acidosis; jf it is due to any other C
Po::.Q:1 roiled Pac~
low HC03 raised HCOJIow
_ • _
Relpirotory acidosis Respiratory oU:olo~i~ Metobolic olkolmi~ Metabolic:: ac::ldalis
Add-hase disturbances can he considered as a set of scales.
Normol acid-bose balance Wht.."11 i1cid--base billance is entirely normal, with no alkalotic or acidotic pressures, it is like haVing a set of scales with no weights on it (Figure 11).
Uncompensated acid-base disturbance VVhen an acidosis or alkalosis deve.lops, the scales become. unbalanC'f'd, leading to acidaemia or alkalaemia respectively. in Figure 121herc is a primary respiratory acidosis with no opposing metabolic process.
3.
ACID--aASE BALANCE: THE BASICS
7\ , /
I
\.
=
"
AClD-8ASE BALANCE· THE BASICS
COMPENSATED ACID-BASE DISTURBANCE As described earlier, a respiratory or metabolic disturbanre is often C9",;>rll5(llro for by i'ldjuslmMt of the utl~ system to offsf1' the primary Jisturbiln..:c. Figures 13 and 14 represent two St.'Cnarios in which the hU1~ have responded to a primary metabolJc aad~is by increasing alveolar ventilahon to chminalC more CO2 (compensatory respiratory aJkillo<;i<;,). In Figure 13 an acidaemia pE'rsic;ts despite u.IInpcns;!tion (partial compmsation); In Figure 14, blood I'll has rrtumed tu the norm.u r.lngc (full cOffipens;ttion). Whpn facffi with ~uch lUli\BC, how can
disturbance and which is
wp
t~ rompcnsalOry
tell wru<:h iSlhe primary pf'OCt"
The fir.,t rule 10 remember is that llwJcumprnSlltion dots not (ICCIlr. The midpOint of the acid----basc scales lies at a pH of 7.4 (H' 40). If the scales tip loward ilCIdaemla (pH < 7.4), this <;,uggests a primary acit.lotk process, it they tip mward alkalacmia (I'll> 7.4), a primary alkalotic pruc(,::>s is likely. The 5('('ond rule 15 that the patirnt is /IIQre imporltlnlthnn lhe ABC. When considering an ABG, one must always lake acrount of the clinical ('\.IIltexl For exampw, if the pabent in Figure 14 W~ diabetic, with hIgh levt>ls of lc.ctoncs in the unne, It would be obvious thai Ih... melabolic acid~ls was a primary pJ"O("~ (diabetic ketuadJosis).
It ;$ not ~ «)$Y kl ~ two pnmory oppoo$Ong p«X:flM$ from a «WTlperuoted d,w.bonce ;. more pr«:iMl
method Ihon !hot de~
obo.... involvM cakvlating Ihe ellpoc.oo compensatory response lor ony g,ven primary di1turbonce. Howevtlr the~ colculations are usually unneceuaryond ale not required for the case ~r~ in Pan 2.
ACID-BASE BALANCE: THE BASICS
-----,
""""'" """""
rtgUre 14 Fully compemoNJd meIOboIic ocidc)JIJ
33
ACID-BASE BALANCE: THE BASICS
MIXED ACID-SASt DISTUIlBANCt When d primary re.piratory disturbance and primary metabolic di<:turbance occur simultaneously tht.-re IS :.aid to be a minoJ acid--base disturbanct'_
If thoe two
pnx:t.~~ oppose
each other. the pattern will bE'simiiar toa compensated aCld-base dlsturoanre (FigUl\: 14) and the resultin~ pll d('ran~ementwill be minimised. A good@xample c. salicylat~ poisoning. where primary hypeiVentilation (respiratory alkaIMl<:) and metabolic acido:.lS (~hl:ylilte is acidic) occur independently.
On the other hand, if the two processes call~ pH to movt: In tht: sall1~ diro;:tioll (metabolic acidosis and rl:'Spiriltory addos!..:: or metabolic alkaloMs and reo.piral.ory alk.1.1osis), .1 profound acidaentia or alkalaemia may N"iult (Figure 15).
The nomogram An alternative way to analyse ABG~ ~ to usc the acid base nomogram (Figu~ 16). Ky plotting the PIK.u, and H· /pH valut.'S un thcABG IlOJll~ram. most ADGs can hoe analy<;ed. If the plotted pomt lleO oubide tilt: dt.'Sigfl.ltl..'
ACID-8ASE BALANCE: THE BASICS
l'et3tdca. ll NS
Ftgure 15 MIxed respirolory and melobolic ocKlasis.
(H'J-
<'< 1.0 100
5
10
15
"
OJ 1.1
80
30
70
12
80
1.3
SO
"
4C
" "
Nofmal1af9l
30
20 10
0 0
2
0
15
, • •
30
45 80 "'\erial PaC~ (k,Pa)
10
"
"""
OJ """"
Figu,.. 16 The ~rom_
3.
DISORDERS OF ACID-BASE BALANCE METABOLIC ACIDOSIS A metabolic addosis is any pl'Ol.."CSS, otlu:r tlum a rise i1l p~ thai actl 10 lower blood pH. It may occur through accumulation of ffiE'tabolic .:lcid~ (excess mgeshon, 1nrrf'.1"oed production or n>duced. renal excretIOn) or through exct'SSi"'e loss of base (HC~). c.l1culaling the
anion gap (sec over) may help to ~tab~h the caU1Ie uf a metabolic acidosis. Metabolic aridOf;is is recognised on an AI}(; by low HCo, (and negative bas<' exC('5S (BE». TIlcre is normally a rom~tury increase in aly\.'OI.tr vt!1\b1ahun to lower l'at.:0:!. If resprratory com~bonIS
overwhelmed, an acidaf'mia will result. ~.~ty must be judged aN'ording to both thto lmderlying process and the resulhng acidat'mia An J ICO;, < 15 mmol/L (or BE < 10) indiCi1t~ a severe acidotic process whereas ... pH < 7.25 (H~:> 55) constitutes
serious at.idaenua. The dominant symptom in metabolic acidosis is often hyJX"f"entilation (Kussmaul's respiration) owing to the re.piratory compcnsatiun. Otht:r ~Igns are fairly non-speafic or related to thf' wu.lerlying cau~ Pmfound acid:u>mia (pH < 7.15; I'· > 70) may lead to arculal0'1' shod:, ~ dysfuncLion, and, uILimatcly death. Specific caU$CS or mdabolic aadlY.>1S are chscussed in greater detail ill the relevant Casel in Part 2.
36
DISORDERS OF ACID-BASE BALANCE
The anion gap In bkxxl, posilivefy charged ions (cations} musl be balanced by negotivefy chorged ions (onions} 10 maintoin flhtctron8lJlmlily_ Bul when lhe moin colions [Na' + K') are compared with the main onions (et" appears 10 be a shortoge 01 onions or an anion gop.
Anion gap
-+-
HCOJt there
=(NG + K) - (el + HC03) {Nonnal = 10 -
18 mnlOl/L]
The eop b mod.. up of unmeo""red onions wei, u, p1,u>pIKJ'" UIlJ >ulpIKJIt;
and negativefy cholged proteins (these are difficult to measure}.
A raIsed onion gap (> 18 mmol/L) therflfarfl indicates the presence 01 increased unmeasured anions, "9. !octole, solicylalc.
A note on .,. Iodic: ocido!5i!5 This cornman cause 01 melobolic <xidosis <Xcurs whCfl lissues roceive on inooequa", wpply 01 0 1 due 10 e!lh"," hypo.wemia or impaired perfusion. Norma! oelobic metabolIsm {which relies on 0 21 is then reploced by anaerobic metabolism (which generales lactic odd). lactic acidosis;s l!Ierelore 0 markIN 01 tiSSlIe hypoxia ond a useful indicator 01 ~firy in "voriety of conditiOM, ,,,dudir>g \oeVC
37
DISORDERS OF A(lD-BASE 8ALANCE
METABOLIC ALKALOSIS A metabolic alkalosIs Co uuy process, ofh" than II fall ill p",{~. that aets to ulcrcasc blood pH and IS chtlr
of H' ions may imtlate the process but the knlncys have- huge ~ope to correct threatenl"d alkalosi::. by incI'f:'a~ing HC03 excretion. Factors that unpair this rcsporlSf> are therefore also nPCt.'Ssary. TIle usual suspects are depletion of chloridp (01, polaSSium (K·) and !WXlium (Nol') ions - in most C
R.spOllie 01 kidney
, 'f't'Iwl
a- IS n :sto1 ~ \he klcNy ITlU$l Me9 HC~- "
pmerw
eI«:troneIAroI (the ~ balance of positive and negaINe ctages "the bocttl
DISORDERS OF ACIQ.....8ASE BALANCE
SUMMARY Of MEtAIOLlC ACIl>-IASE DISORDERS 10", 1.5.1
Metcabo6c oddosis Uow HCO:s)
,
With ralAH onion gop toclic ocldosi1 (e.g. hypoxoemia, lhod, sep:5i1, infordion)
Ketoacidosis ldiabetes, ~Iorvolion. okohol ex~ssl Renol foihJrft Poiwning {olpinn, melh
With normal an'on gop R:croI tubular «!dom Diorrhoeo Ammonium chloride i~ion Adreool inwfflclClnCy
Vomiting PoIlllsium depletion (e.g. diurebc.1J Cw.iog'~ ~yndrome
Conn', tyndrome !primory hyperokbteroni~J
39
DISORDERS OF ACID
I\Asr BALANCE
RESPIRATORY ACIDOSIS A r....pir.1torv actdo,sL::ll:>, ::,wlpl y, Uti incFt'u:'{' itl Plfl..~. Sinc(' '-- ( )1 dissolves in blood to t(\rm carbomc add, th1::. h<e; tll.,- cUcct oj IU~,"crUl:o<, pH
(fH iClM
~,)rmally, lun~
are ablc to im.:reasc \ Cliliialion to mamtaln • n,'"nnal
in rondlhoTl.
Pz:.
-
('\'f>T1
alvet.,Ia.r \ clltilaliOll Th'
m.1\
occur from ao'! ('all,*, of type Z
I'f'''piratory tmpalTlJlcnt b~ ::;c<;tu.."l 1.3\ ur to CQuntt>fitct a mf'taboh ,11kalosis
RI:SPIRATORY ALKALOSIS A rl>SPllMl1ry ill!.;IlIMI::i III ii n
~~.
,n P
anJ
15 cau~'<.i bv
aJvl'\llar h,,~rvmtilatlon 'rima.... ,U"t'li UKlude. Dalll, an.'Ilcl h) pen ,,·lltil<1.1101l syndwme i(>Vt"f b'e;lthk>AAnp<:<; and hypo'l.M:"ru.1 It mav '1lso occur to IOUIlb:ral,;
Ac:u~ ven.us (hroni<
ao
U\
respiratory CKido~.
_
SIflC& metoboitc compenloOtOfy ''-''''fX.''"'Cs l<J~e dnp 10 develop, ocule re~;rotory oc,do,i,
'olmos! ofwoys
utlcompenw/<..'Ci.
l.....ditl(l rnnidly
lu profo<Jnd (md don.g&rous acidaemio, lndeed, on opposing m<.-1uboli, olkoloSIS wggesh Ihol u rl'sp;mlory q
for some time. In other wOld~. the fl\C>(:IIC(·
,,/
mptaoolic compen$OtlM
di$ling<JIsMs chronic from o<:Ule Iy~ 2 -venh!otory IUllme j,.".. sedi"," I 3}
40
DISORDERS Of
AC~ASE BAlANCE
This is the must dang~rous pattern of add-base abnormality. 11 leads to profound acidac:mia as th~t'E' Me two simultaneou!I acidotic processes with no compensation. In clinical practice it is often due to severe ventilatory failure', in which the rising "1lCOz (respiratory acidosis) i:. accompanied by a luw PiIOJJ ff'Sulting in Lissue hypoxia amI consequt'nt L1.clic acidosis.
..
ABG SAMPLING TECHNIQUE Before you can interpret an ADG you must, of rour::.e, obtain a :.ample of arlen",1 blood.. 'llie {ollowin,; steps should be ~ as a gwde but the best way to Ipam is at the bedside with expf'nenced
supervision.
BEFORE SAMPLING • Confirm the 1It.'t..>d for the A tjG and identify any contraindkations (Box 1.6.1). •
Always record d(>tails of O:!: therapy and respiratory support (e_g. ventilator settings)
•
Unless results are requi.n.'d urgently. allow alleasl20 minute'S after any change in ~ tlu:rapy before samphng (to achieve a steady state).
•
Explam 10 the patient why you 3r'f" domg the test, what it involvtS and ~ possible complications (bleoeding.. bnusmg, arterial thrombosis, infection and pain); then obtain consent to proceed.
•
Prepare the m..~ equipment (hepari.niscd syringe with cap, 2D-22G needle, ::.harps disposal container, gauze) and don universal precautions.
•
Identify a suitable site for sampling by pUlpilling the radial. br.lchial or femo'd.l artery (Figurf' HI). Rouhne sampling should, initially. be attempted from the radial artery of the non-dominant arm.
02
ABO SAMPUNG TECHNIQUE
:"C"-...........
8
A
rlgU"e
18 At~roJ pvnclure sife5
lox 1.6.1
Conh'OindicaticMutoAaG~si,·
Inodequote collolerol ci,culollon ollhe punetut. 5ite Should not be perform~ through a Ie~on or a 5Ufgicol shunt Evidence of periphllH"al vosculor dl.eoH dislontlo the PUrl(:tuf8 sile Acoogulopoltly or medillm- to h;gh.do~ anticoogulation theropy
ABG SAMPliNG TECHNIQUE
RADIAL ARTERY SAMPLING •
Pt:rform a modified Allen test to ensure adequab> ('oliMeral circulation from ulnM arteryl (rigure 20).
• Position the p,ltknl's h
Identify the radial artery by palpatinl'; the pulse; choose a site where the pulse is prominent.
•
C1~an
•
EXf*1 th.. heparin from syringe.
the sampling site with an alcohol wipe.
• Ste,ldy your hand on tile patient's hand, as showlI, the n~le at 45°, bevel facing up.
•
th~n
insert
Be surf> 10 insprt the need Ie slowly to minimise the risk of arterial spasm.
• When the llt.wle is in the artery a flash of pulsatile blood will appear in the barrel of the needle. Most ABC syringes will th('tl fill under arterial pr"'SSllr(> (see info box over page). •
Obtain at least 3 UlL of blood before withdrawing.
'However, the value of routinely pedorming a modified Allen's test prior to arterial puncture has been questioned, in part due to its poor SP1lSitivity and specificity for identifying inade.l' ,atE' col1aleral cin:ulalion. (Slogoff S, Kc-"ts AS, Arlund C. On the s..... fcty of cadi"l artL'rY (;;lJUlul"tiOIl. An"esthesioloj;\Y 1983;59:42---47). 44
ABG SAMPLING TECHNIQUE
ngure 19 PO$jtioning 01 _i511ot A8G sompIing.
Arleriol sampling !porticvlor/y from the rodiol ortery) can be extremely pain/vI; dlscomforf can be reduced by inj.cring 1 ml 011 % lidocaine, at !he needle inMtrtion ~ite priOJ 10 wmpling
Venous _ &16.iaI lilac II?
Ooft. non.pul,otil. blood that requirti monuoI wction 10 os.pimle often inctocole$ a Y8OOt.I$ ~ ~ In _ e modVcOl'dioc orr.g). AnoIher dve " when ~ on A8G OldywS 's ''9n~1!y lower than 500: on pulse Wtirnelry.
.,
ABG SAMPliNG TfCHNIQUE
AFTER SAMPLING
•
Once duequatc blood has bft>n obtamed, remo,'c the needle and apply firm, d1rect pn.'S:iourc to the samplp ~lle for at least 5 minutes (and until bl~ing has cea:x'd).
•
lliposc of all sharps and contammated matcrials approprialply.
•
l'nsHrt> that no air bubbles are present in the samph~, as thcy may compromise results. Any sample with more than very fili~ bubbk--s should be discard<>:d.
• The sample Iohould be ;lnaly~ promptly: if the transit lime is likely 10 exl'ffd 10 minute'S, then the syringf' c;hould be blon.'\.! on crushed ice, •
..
If sampling IS WlSu«cssfuJ it is often advbablc 10 repeat lhe test on the OPPOSite wnst ib e\'Cfl sliW11 irritation of the artery on the first
I
ABG SAMPLING TECHNIQUE
,
InWuct h patieQ 10 dlnctlliIitIIt fist !hen oc:cllde bJltl;wlenes
•
--
'--LJ. Rerllll .,. .... ar1efy.
c
• fit ooIow IllUns to .. r.1dMllW'l1Clllhishi ?m . . , , I m:wtion
figure 20 M.ocJif,ed Allen test. 47
WHEN AND WHY IS AN ABG REQUIRED? 1. TO ESTABLISH A DIAGNOSIS The- ABc. i~ integral to the diagnosis of respiTo1tnry failure and primary hypt>n'pntilahon. It also u.lltlllifics the pn>sencp of metabolic lkidosis and alkalosis.
2. TO ASSESS ILLNESS SEVERITY Thp four ABC valUel> l>hown in the t.1ble bl:>low, In aJditiun to owrall clinical QSHSSmm/, may help id\;~lIti{y critically \lnwpl1 patients rt:y,uiring Llq~ent inteTVf>ntion. Pao, < 8 kPa
"'0,
Bdow 8 fo{l~ in Pro, produce a marked redudion in Sox>, Ithe 'sleep' port of 1M 0, dl$KIClOlion curvel and hence 0, content 01 bbod Rising P0c0 2 Since .eool compem.ation occurs ewer dayJ 10 weeks,
ocu~
riS&s in PocO]
prodoce 0 cOf.espooding drop in pH. In respiratory distress, rising POCO] often signifies exhaustion and
,e:ve!$01 of !he
-
proefl~,
at: < -10/HC01
c
15
on omil'lOl./S s'gn. Polienls require Ulgen!
Ieoding to ven"lotory foilure or auisted venhlahon
15
This ....:rtu. is included in -..01 ~_...Iy 5COring syVelns arK!, when d""" kl Iodio: ocide»is, indicates 5e\Iere hypoluO of cenuktr IeYeI H+ > 55/pH
<
7.25
A ~i9llilicant decrease in pH ovl$ide the nOHoal ra"9fl indicates that compeosalory ~ir.ms have b.en overwhelmed and i$ a medical emefgeoc:y
••
I
WHEN AND WHY IS AN ABG REQUIRED?
3. TO GUIDE AND MONITOR TREATMENT Rf'&uliU" ABC momtoring can help provide early warning of do;>lcriorabun and jl1d~e the cffe<:tivl;>ness of therapeutic inlprventiol\:>, It i:o essential for tilrating 0, therapy in patients with chronic tyl't! 2 rt'Spiratory f.lilurc and for optimising \'enhlator YffinKS.
Box 1.7.1
Clinkal scenorios In which an ABO is us.ful
Establishing dtagnosb ond aSMsstng i1lneu MYel ilt 5u~ hypercoptiio
It PocoJ
Drowsiness, Roppmg Iremol', bounding pvI:IC~ Clinkol dete.iorotion in polient with chrOflic type 2 re$.pirOiOry impoirm.nl Of' prediipO$ing condItion le.g. chronic ob~ud1Ve pulmonory diseas.e\ Suspeded severe hypoxoemlO Very low Of unrecordable 0, saturaTion; CYOrlO$i~ Severe. prolonged or WOfSflning m,plrQlory distress Smok.e Inhalotion Ica.boxyhuemoglobln level) Hypervenlilonon (confirm I P~, check kw- UIldertying m.lOboIic ocidoslsl At,. detel'lOl'ohon III consciousne!oS Any leVu.ely unweH potient' Pulso oximetry unreliable 01 $U~piclouS re~ull As port of a 'A<:ogmsed Ilinen severity scoring ~y,tem le.g. Glosgow cri...in in poncreohhsl
Guiding II ealli"'" and
~itonng respon5e
-
Mechanically ventiloted JXItients Potients receiVing non-Invosive o"i,ted vcntilolton Patients w,1t. respiratory failure Potlents with c:hroni<: hyperCQpmo receiving ~ Ctiticolty ill polients undergoing surgery Candidote, lor long-term olt)"96n the,apy "Inc:koding, bu4 ,101 ."",1dIId lo, '''0(1. ~psi., buo n., mol'" 1raun'Ia. ocute obdomcn, poisCO'Iing, ~oocl;ocJrenol/hepo1i<: k>iIur.. JootJe• •eaoocidosi.
....
.9
COMMON ABG VALUES The followin/o; parameters arc commonly found on ABC reports and
ar", proviJ~ for wC",rence (normal ranges in btadcts): W 135-45 nmol/ll < 35 _ olkoloemio, > 45 = ocidoemio Concentration of free hydrogen ions: lhis is a measure of how addle or i1lkalinc a ~lution IS.
pH (7 35-7.45)
<
7 35. ocidoen'llo, > 7.45 .. olkaloemio
Neg.1.tive 10K of the H' ion concentration: this is a common representiltion of th", H' I.VI'l(ltfltr.. tion. Due to !.he logarilhmic nature uf the :Kale, !lOmaU changes in the pH actually represent largt' changes in the H- C'OJl("M\tration
f'O:ll> 10.6 kPo or > 80 mmHg in arterial bbod on room oirl rdrtial pres:;;ure of~: can be thought of as ~ drive for 0:1: mokculcs to move from onE' pl3cp tn anothM'. I'O:t L'l not a measure of Oz COfltent tmt it does dett'flJline the extent to which Hb is sahJrated with~. p~ refers specifically 10 the partial pressure of Oz in arlerW blood. ~ 14 7-60 kpo Of 35-45 mrnHg in arterial blood)
Partial pressure of C~: can be lhoughl of as the drive for Co,; molecu I~ to muve from ~ place to another and (unlike Po;) is directly proportional to the amount of C~ m blood. pOCO:! refers specifiCAlly to the partiaJ pressure of ~ in arlmlll blood.
5o:l1> 96% on room oir)
0 1 saturation of haemoglobin: the ~tage of o,.-binding site> on lib proteins occupied by ~ molecules. This is a measure of how much of lhc blood's ~-earrying capacity is being used. Sao-l refers ~ifical1y to the 0" saturation of rJrtmol blood.
,.
COMMON VALUES AND THEIR MEANINGS
HCOJocl (22-28 rnmol/ll Actual bicarbonate: the plasma bicarbon.:ttc concentratiun caiclilutcd from the actual pe02 allu pH JIlement of hirarh0natp in A Kt--; analysis.
BE (-2 10 +2l Base exre::;s; a calculation of the amount of base that nP1:'ds to be added to, or removed from, a sample of blood to achieve a neutral
pH, at 37°, after PeOz/las been cortt'Cled 10 5.3 KPa (40 mmHg), A positive BE inJicates that there i::; more base than normal (metabolic alkalosis) and a negative BE mdicates that there is l~ss base than normal (ml2'tabolic acidosis). ladate (0.4-1.5 mmol/l) An indirect measure of lactic acid: high levels of lactic arid are a sign of tissue hypoxia.
Hb n3-18 g/dl men, 11.5-16 g/dl women) Pla,5ma hapmoglobin concentration: this effectively determines blood's capacity to carry O 2 , Na 1135-145 mmol/LJ Plasma sodium conc~ntration, K(3.5-5 mmol/ll Plasma potassium concentration. Cl WS-:-l 05 mmalll) Plasma chlorid~ concentration. iCa {I.(}-l .25 mmol/l) Plasma ionised calciwn concentration. Glucose (3.5-5.5 mmol/l if fosting) Plasma glucose concentration. 50
MAKING ABG INTERPRETATION EASY The golden rule for making ABC interpretation easy is to assess pulmonary gas exchange and acid base status
Tobie 1.9.1
p~,
Sao,
Mild
8-10.6 IJ'Q 60-79 mmHg
90-94%
Moderate
5.3-7.9 kPa 40-59 mmHg
75-89%
Severe
< 5.3 kPc < 40 mmHg
<
independently.
"AS$,n~G:"PlLQiiO~4YqA'S .!'~Il~i'J~ • Using the algorithm opposite, classify gas exchange into one of the four possible categories.
• If there is type 1 respirator], impairment, assess severity of hypoxaernia (box 1)
• •
If there is type 2 respiratory impairment, establish whether it is chronic or acute, then assess severity of hypercapnia and hypoxaemia (boxes 1 & 2)
If the (
An.Sling hypoxaemia ieverity
75%
or: High fiO~ requirements to maintain odeqoote Poo~
;~~)'''11
.
0~" n~f~; "'~" ,'".- ' ectp, •. -, .- . " .. ,,< .",
Severity is nol reloleo:::! 10 abr.olule Po::~ "ollie but 10 Ihe raie of PacOj rise and degree of blood pH derongemllnt (pH < 7.25 • $evere acidaemia). The presenctl of exi>a...stion is olso on o'llinous sign.
..
I
MAKING ABG INTERPRETATION EASY
~
_ _ ~ _.::..'i- . .
"
••
INTERPRETING ACID·IASE STATUS •
Usc the flow chart opposite to broadl}' classify acid-base status. • If the patient has a metabolic acidosis, calculate the anion gap to narrow dov.'Tl the differential diagnosis. Anion Gap"" (Na + K) - (el + HC03) Normal
=10 - 18mmol/l
• U the precise acid-base derangement is not immOOiately clear (ego middle column) then remember the
followmg points: Always consider the clinical conlexl when interpreting acid-base status. Metabolic compensation takes days 10 occur, re>piratory compensaoon takes minutes. Overcompensation does not occur. An apparent compensatory response could repl"e5ent an opposing primary process. •
Note that with a very mild acidaemia or alkalacmia both PaC0 2 and HCO) may be just within the "normal range" (considered to be a mIld mixed aad-base dISturbance).
-,-
PlCD,- 4, 7-U kPiI or
~5mm1-1g
BEj-llOo2j ~11 (22-28 ~
pH (7.35-7 45)
If' (35-45 Md4)
~ ~ . z
~ >
§ ~
MAKING A!G INTERPRETATION EASY
••
APPENDIX 1
The A-o erodi«llillhe difference Dtt.o.een !he ~ in oNeoIi IPAo"l ond the Po, in ortef"ioI biood lPa011. p~ is meowred Of! A6G bur PAc" hos to be calculated us.ng !be oIveoIor gc» equation 1- box beIowj. A-
G
flruclien' • f1'~ - ,..."
1111 noflnolly bs thon 2.6 i.:Po (20 mmHg). oIIhough it increo~ with oge
and F~, This meam Ihat: I. the l'lOl'moI range fOf Poo, IoUs wilh oge 2 me A-o grodient is most accurate when performed on room air
PAo, (kPo)
= (1;0, • 93.1) - (Poco, • 1.2) ~
Neo:2 ImmHg) = l'iD, • 713) -
5.
(I'ato, .. 1.2)
THE ABG IN PRACTICE CASE 1
History A 25-ye-ar-01d man, with no significant past medical ~toI)', presents to the emergency de-partment with a 2-day history of fever. productive rough and woT5elling breathlessness.
Examination He is hot and flushed with a temperature of39.3"C. He does not appear distressed but is using accessory muscles of respiration. There is diminished chest expansion on the left with dullness to percussion, bronchial breathing and coarse crackles in the left lower zone posteriorly.
""'"
lte:5pirotory role BIoodpn=~ ~ lroom airj
58
HM beats/min 28 breaths/min 118/70 mmHg
"""
Arleriol blood go~ 23/7/2006 Unit no,: 00654545
On room air
ID- John Simpson
No<~1
W
pH 1'00,
Po,
..... "
SPa, locto.. K No
CI ICO'
Hb Glucose
31.8 nmol/L 7.50 3.74 kPo 28.1 IT\rTlHg 768lcPo 57.8 mmHg 23.9 mmol/l -0,5 ml"lOi/l 88.7~
12 37 mmc1/l
138 mlTlOljl 99 mmol/l 1.2 mmol/l 15g/dl 5.4 mrnol/l
{35-451 (735-745} (47-6·01 (3545)
1> 10.6}
t> IlOl (22 28} 1-2_2) 1>9n) 10.4-1 5} {3.5-51 1135-145} (95-105} {l-1.25)
[13-18) (3,5-5.5]
Questions 1. a) Describe his gas exchange.
b) Describe his acid-base status. L Should the patient receive supplemental O:z?
J. Is pulse oximetry a suitable alternative to repeated ABG
mOnitoring in this case?
,.
THE A8G IN PRACTICE
CASE 2
History 1\ 34-ye,1f-old morbidly obese female with a body mass index of 49
has an ABC sample taken as part of her prroperatiYe assessment for weight reduction surgery. Apart from morbid obesity and type 2 diabetes, she is otherwise w and has no ~piratory symptoms.
60
THE A8G IN PRACilCE Al1eriol bbad 901 14/01/2005 Uril no.: 553Q208e
Gnat.-
10: Morcella P1antogenet
""'mal H"
pH Ito,
Bicarb
" ,'odaM
45 nmol/l 7.35 7.3lPo 54.8 mmHg 9.6 i:.Po 72.2 mmHg 29 mmol/l. .,).8
Sib,
96"
No
I 4.7 mmol/l 134 mmol/l 102 mmoVl , 2 mmol/t
cr
u· Hb GIl)cXlSEl
13g/d1. 9 mmol!l
135-451 [7.35-7.45) (4J-6.0I
(3'-45} I> 10.61 I> 80) (22-281 1-2-+2) (> 98'%) (0.4 - 1.5]
13.5-5) 1135-1451
19S-105J 11-1.25) (13-18j 13.5-5.51
Questions
t. a) Describe her gas exchange. b) Describe her acid-base status. 2. "''hat is the most likely diagnosis?
61
THE ABO IN PRACTICE
CASE 3
History A 24-year-old female nursing student attends hospital complaining of sudden-onset breathlessness. She flew to the UK from Australia the previous day and is very concerned she may have a pulmonary embolism. She has no pleuritic pain, haemopt}'sis or leg swelling. no past hisrory of lung disease or deep-vein thrombosis and is a non-smoker.
Examination She appears anxious and distressed. Her respiratory rate is elevated but eMt examination is unremarkable and there are no clinical si of deep-vein thrombosis. A chest X-ray reveals no abnonnalities. p,,~
Respirotory rote B600d preuure ~ lroomotrj
62
8e beals/min 22 breaths/min 124/76 rflmHg
95"
THE ABG IN PRACTICE
23f7/2006
u""
no.: 00654.545
10: Jell Royds Normal
eo
J l.2 nmol/l 751 3.90 kf'iJ
135-451
293 m."Hg
10.3 kilo
135-451 f> 10.61
77.0mmHg
(>
Bicorb
25.0 mmoVt
{22-28)
BE 5po, lodoM
1-2-+2,
,
+0.7 mmol/l 93.7% 1.0 4.3 rnmoI{t
No
141
d
101 mmol/l. 1,2 mmol/l
fl35-145) (95-1051 {1-1.251
13 a/dL
(l3-1BI
4.6 mmol/l
(3.5-5-5)
pH
"'o,
m<nol/l
(7.35-7.45)
1A.7-¢·0I
(> 98%) (0." - 1.51
13.5--51
QueSt101U
1. a) Describe her gas exchange.
b) Describe her actd-base status. 2. \\'hat is the likeliest diagnosis?
801
THE A8G IN PRACTICE
CASE 4
History A 78-year-old male on a surgical ward is found unresponsi\'e, having returned, a few hours previously, from a complicated open cholecysteclomv.
Re...iew of his charts J'e\'eals he has recetved three 10 mg injections morphine since returning to the ward, in addition to the morphine delivered by his patient-controUed analgesia device. Examination
The patient is unresponsive with ~llow respirations and bilateral pinpoint ptJpils. PlIlsll role Respirolorl' role
90 beob/min 5 breatlu/mirl
98/64 mmHg
8Iood preullle 500,
....
8M
5.6 mmol/l
u
..
i
THE ABG IN PRACTlCE
,Ivftwel blood gos 18/09/2006 l}r1it (lCl.: 6799986 [l Henry Spc~~
".
pH
"'"
""
Bicarb
"
,"'"
~No
a ;Co'
'*'
Glucose
On 28%0"
Normel 65.4 nmcl/l 7.18 B.2 ~Pe 62 mmHg 11.76 kJ'e 87 mmHg 22.4 mmoVL -1.5 mmol/l 99.8~
1
4.4 mmol/l 137 mmol/l 103 mmoI/L 1.16 mmeVl 11 g/dl 3.9 mmol/L
{35-45/
(7.35-7.45) (47...o.0J {35-451
I> 10 61 [> SOl
[22-28)
1-2-+21 (> 98'l.j 10..4-1 5) p.5-5J 1135-1451 (95-1051
11-1.251 !I3-181 (3.5-5.5)
Questions
1. a) De$(:ribe his gas exchange. b) Describe his acid-base status.
t What IS the m06t Wc:ely diagnosis? 1 What treatment does this patient require?
ThE ABG IN PRACTICE
CASE 5
History A 75-year-old man IS brought into the emergency department by his family. He is extremely short of breath and struggling to speak. Following a conversation wilh his family it emerges that he has a long history of chronic obstructi\'C pulmonary disease.
o.'Cf the lasl 3 day'S his breathing has worsened consideTably and he has expectorated increased \'oIumes of sputum.
Examination The patient is struggling for breath and appears extremely dis,lre". He exhibits signs of chest h)-pcrinflation and is breathing through pursed lips. Breath sounds are generally diminished but there are added sounds.
PulMi
120 beals/min
Re~rak:Hy JU'tI
26 bnlolls/min
8Iaod preuure
150/80 ll'ImHg
Temperature
36·C
Sa0 2%
'I"
..
THE A8G IN PRACTICE Arterial blood gas 10/07/2005 Unit no.: 77655349 ID: Joseph Spielmol1
H' pH Pto"
Po, Bicorb BE SPO, lactate K
No CI iCa'
Hb Glxose
39.5 tlmol/l 7.40 4.9 kPo 36 mrnHg 5.6 kPo 44 mmHg 23 mmol/l -1.2 mmol/l 80% 1.0 4.1 mmol!l 137 mmol/l 99 mmol/l 1.1 mmoljl 16.5g/dl 3.8 mmol/l
On oir
Norma! (35-45) [7.35-7.45) [4.7-6·01 (35-45) [;. 10.6J [> aOJ [22-2a} {-2-+21 I;. 98%) {O.4-1.5] (3.5--5)
(135--145J {95~105]
(1-1.25) (13-18) (J.N.51
Que5tions
1. a) Describe his gas exchange. b) Describe his acid-base status. 2. Should you provide him with oxygen?
.7
THE ABO IN PRJI.cllCE
CASt 6 History
The pabent from the pre"ious case is treated in the emergency department with nebuliscd bronchodibtors, oral prednisolone and antibiotics. He is then transfened to a respiratory ward where he it administered 28% oxygen by a fixed concentration mask. Despite this, his Sa0 2 (as measured by pulse oximetry) increases only marginally and there is no imprO\'ement in his symptoms. Examination
Examination findings in the chest are unchanged but he now appoool emausted and slightly confused.
--- - - ---
Re~loIory mle
120 beats/min 16 breath~/mln
8bxl pressure SoO,
120/80mmHg 83'X.1on 28% Q,]
Temp.... alure
Moe
Pulle
-
A repe.1t ABG is performed (6 hours after the first ABG).
THE ABO IN PRACTICE ,ArMori'" b£ood gas 11/07/2005 Unit no.: 77655349 10 joseph Spielman
On 28% OJ
N~ma1
rwnoI/l
W
51.4
oM
7.29 6.9 kPo
"'" II>,
-'" "
511>, lactate K No d
..."".
Gtuco~e
52 mmHg 6..4 kPo 48 IfImHg 24 mmot!1. -0.9 mnol/l
....
1.0 4.0 mmol/l 137 mmol/l. 99 mmai/l 1.1 mmol/L 16.5 g/dl 4.2 mmol/l
13>-<15) 17·35-7.451 14.7~·0l
13>-<15} {> 10.61 (> 80) f22-281 1-2-+2} I> 98'%) (0.4-1.51 (3·5-51 (135-I45J (95-105) 11-1.25) 113-181
13.5-5.5}
Questions
l.a} Describe his gas exchange. b) Describe his acid--bi'lsc ~t"tus. 2. Should his oxygen now be stopped?
••
THE ABG IN PRAcnc:e
CASE 7
History A 77-year-old woman is admitted to the stroke ward with right-sid weakness, visual disturbance and slurred speech. She is commenced on nasogastric feeding due to swalJO\\ing problems but has a large vornlt 24 hours later. She initially appeal'S well but oyer the next few hours develops .....orsening breathing difficulties. Examination
She is agitated, distressed and pyrexial. A dull percussion note and coarse crackles are evident at both lung bases. Other than acute confusion, neuroIogkal findings are unchanged from admission. Pul$e Re$j>irolory ro)e
Blood preUl,,Ire SaO, '''''' 0,)
70
92 beob/mln 28 breotlu/min 112/65 mmHg
.,,,
THE ABG IN PRACTICE ·Merial blood go$
On 60"4 O 2
23/7/2006 Uril no.: 00654545
II). Mrory Woteo
No""" W
38.8 nlrlOllt
f.l5-45J
pi<
7.41
"'..0,
4.43 lJ'o
(7.35-7.451 (4.7-<10)
33.2 mmHg
135-451
8.67 kpo
(> 10.6) (> 80)
Ib, Bico,b
" •
SAo,
loctcle
No C iCo·
Hb
G1"'COH
65.0 mmHg 21,2 mmol/l -2.8 mmol/L 927% 1.6 4.0 mmoi/L 144 mmoVL 103 mmollt 1.2 mmol/L 13 g/d!. 6.6 mmol/L
[22-281 1-2-+2)
I> 98%1 (OA-I.5)
13.5-61 (135-145) (95-1051 11-1.25) (13-18)
[35-5.51
Questions 1. ,I) De;eribe her gas exchange. b) De;eribe her acid-base status.
2. What is the most likely diagnosis?
3. ~ her condition mild, moderate or se...ere?
71
THE AEIG IN PRACTICE
CASE 8
History A 68-yt'ilr-Qld man with chronic obstructive pulmonary disease is referred 10 hospital by hIS doctQr with a short hislory of increased breathJeossness and noduced effort tolerance. He is nonnall.}' ca of walking around 500 metres but now has difficulty dressing a breathless at rest.
Examination He is lucid, alert and nuJdly distressed. He is using acnsscry muscles of respiration and breathing through pursed lips. Chest examination reveals features of hyperinlliition, generally dimin' breath sounds and scattered rhonchi. (wheeze).
""~ ResptrolOry role Blood prllmH8 SaO, (100Il'I oir)
96 beob/min 24 breotbs/min 13B/82 mmHg 78%
,
THE ABO IN PRACTICE Arterial blood gos 23(7/2006 Ullil 110.: 00654545 tD: Hamish Roy
W pH
Pco, Po,
8icarb BE SPo, (octale K
No CI iCo'
Hb Glucose
43.2 nmoljl 7.36 7.20 kPo 54.1 mmHg 5.3 kPo 40 mmHg 30.6 mmolfl +4.9 mrooI/l 75.2% 1.2 3.7 mmol/l 144 mmolfl 102 mrnol/L J.2 mmol/l J6g/dt 4.9 mmol/l
On air
Normol 135-451 (7.35-7.451 (4.7-6.D) 135-451 I> 10.6) (> 80) (22-28] (-2-+2) (> 98i~) [0.4-1.5) 13·5-5) {I 35-145) 195-105) [1-1.25) {\3-18) 13·5-5.5)
Questions
1.3) [)es(ribe his gas exchange. b) Describe his acid--base status. 20 Which one of the
foUowmg ABC values is most likely to have changed significantly in the past 24 hours; pH, PC0l-- 1'0:2 or HeO)?
3. \'I,1tich two of the above ABC values indicate the need for caution when providing O 2 therapy?
73
THE ABG IN PRACTICE
CASf9
History The patient from the previOlis case is treated with nebuliscd bronchodilators, oral prednisolone and 6O'Y.. 0: by face mask. His oxygen saturations improve significantly but when he is revie\>ved 1 hour later, his condition has deteriorated
Examination He is drowsy and barely rousable. He no longer appears to be in respiratory distress and his respiratory rate has fallen to 14 breaths/min. Olest examination is unchanged from pre,'iously.
p~~
Il:e$pirotory rote
Blood ........ ~
,.
(room air)
88 beoh;/min I. breaths/mm 132/80 mr'IHg 96..
THE ABG IN PRACOCE Arferiol blood gas
0..60%0,
23/7/2006 \,kIit 00.: 00654545 ID Homish Roy
pH
50.8 nmollt 7.29
Nonool 135-45} {l.35-7.45)
I
8.7 lrJ'o
(4.7-6,0)
65.3 mmHg 11.2 \'Pc
(35--451 I> 10.6} t> 80)
...
""
"""b
84.0 tr'IllIHg 30.3 mrnol/l.
f2'-"0I
....7 mtne>1/l
....2-+21
SIb,
96.2~
"""' K "
I2
f> 98'%1 10.4--1.51 {3.5-51 (135--1.45) (95-1051 (1-1.251 11J.-18) (3.5--5.51
"
No CI iCa' Hb (;1,-
3,6 mmol/l 14.4 mrnol/l 102 mmol/l 1.2 mmol/1 16 g/dl 5.0 mrnol/l
Questions 1. a) Dl-scri.be his gas exchange. b) Describe his add--base status. 2. What has been thecausc of his deterioration?
THE ABG IN PRACTiCE
CASE 10 History A 21-year-old woman presents to the emergency department with a 6-hour history of worsening breathlessness and wheeze. She has a past history of asthma with two previous exacerbations requiring hospital admission. She now feels very breathless and is obtaining relief from her salbutamol inhaler. Examination
She is tachypnoeic at 30 breaths!min, is using her accessory muscles of respiration and only just managing to speak in full sentences. Auscultation of her chest reveals widespread polyphonic wheeze.
Pul~e
Blood pressure
115 beats/min 120!80 mmHg
SrD,
96% lroom airj
Peak elCpiralar)'
76
flow
160 LIs lpredicted. 400 lis)
THE ABG IN PRACTICE ,Artenol blood gas
OnD"
12/11/2005 U,nl no.. 126399...3 ;0 Jessica Gokhon
tr pH
"''''
FO,
BI~orb
BE 500,
,
lado" ~o
a
I ,Co' Hb Glucose
42 nmoI/L 7.38 5.8 U>o
"""mol 135-45) 17.35-7..45) (....7-6.01
43~Hg
135-451
10.2 kPo 76 mmHg 24 mmollL -1.3 mmol/l
I> 10.61 I> 80)
122-281 1-2-21
96"
I> 98%1
1
10.4 - 1.51 (3.5-51 1135-1.451
4.0 mmol/l 140 mmol/l. 99 mmol/l I 12 ",maI/l 130 g/dL 5 mmol/l
(95-1051
11-1251 P 1.5-15·.51 (3 5-5.51
Questions
1. a) Describe her gas exchange. b) Describe her add-base status.
2. Which of the above ABC values gives the greatest cause for concern? 3. flow would ,·ou classify the seH''I"ity of this asthma attack?
77
THE ABG IN PRACTICE
CASE 11
History A 23-year-old woman presents to the emergency department willi .. painful ankle, following a simple trip. While being examined by the emergency department doctor she becomes extremely agitated and upset. Despite a nonnal ankle X-ray and extensive reassurance by the emergency departm€I1t staff, she refuses to believe that her an is not broken and staTts crying. V\-1l.i1e leaving the d('partment, she develops a dutching: sensation in her chest, shortness of breath and a tingling sensation in her hands and around her mouth. She ll.:'pO that she feels unable to take a deep breath. Examination
The patient appears frightened and extr\;.'tllely distn>sscd. Other than t1lchypnoea and a mild sinus tachycardia, cardiorespiratory examination is unremarkable. Electrocardiogram, chest X-ray and peak tlow measurements are all normal. Pulr.e I<:es.pirotory rate Blood pressure
s,D,
7.
96 beats/min 36 breaths/min 130/BO mmHg
100%
THE ABG IN PRACTICE Medal bk>od gas
Oneir
12/11/2006 l)roil no.: 12534943 !O Tril'lllY Forqhuof
... pH Ito,
29 nmolfL 7.53 3.1HPo
Ib,
24 mmHg 1".3 ~Po
Bicorb
108 mmHg 24 mmol/L -1.8 mmol/L
No<mal (35-45) {7.35-1.451 (.4 7-6.01 135-45) I> 10.61 I> BO) 122-2BI
BE 500,
99%
1-2-+21 (> 98%)
eoc-
1
10·4-1.51
<
3.5 mmol/L 140 mmol/l 99 mmol/L 0.9 mmoJjt
13.5-5)
No
CI iCo'
"b "'OC~
12·°9/& 5 ntmol/l
1135-145} (95-105) (1-1.251 (I1.5-15.5)
13.5-55)
Questions 1. a) Describe her gas exchange. b) Describe her ucid-base status. 2. Are there any other abnormalities?
3. What is the likely
diagn~is?
79
THE A8G IN PiACTlCE
CASE 12 History
A 4O-Yl'ar-old male is pulled from a house fire and brought 10 the emergency dt-partment in the early hours of the morning:. The pilrillnedics estim."lte he is likely to have been trapped in a sn\oke-filk--d room for up to 20 minutes prior to rescue.
Examination The patient is heavily contaminated with soot and smells strongly of smoke, Fortunately he has not sustamed any thermaj injuries, He appears to be confused and has just vomIted. Baseline observations are nonnal, with an oxygen saturation of 99% on 15 L Oz by rn."l.<;k.
80
THE ABG IN PRACTICE Arterial blood gos 10/08/2005 U'lIIlO.: 77634566 1D: Robert Jooes
On 15 L O 2 by mask
Normal
W
pH ",0,
Bicorb
BE 51'>, tac!l:rle K No
CI 0C0'
Hb Glucose
414 nmol!l 7.36 4.5 kPn )4 mmHg 47 "'" 353 mmHg 18 mmol/l -5.5 mmol/l
low. 2 3.0 mmol/l 145 mmol/l 103 mmoI/l 1 1 mnoI/l
14 9/& .4 mmoi/l
(35-4.51 (7.35-745) (47-6.0) (35-45) I> 10.61 I> 801 (22-281 {-2--t21 (> 98'%} (0.4-1.51 p.5-5) (135-1451 {95-105) (1-1.25; {lJ-1SI 13.5-5.51
Carbon monoxide G5SCIy CO
40%
[non-smokers < 3%1 {smokers < 10%1
Questions 1. ,1) Describe his gas exchange. b) Describe hlS acid---base status. 2. \Vhat is the mosllikcly diagnosis? l. Which of the values pro\"ided isfDls.·fy high: POlOz, ~ or Hb?
&l
THE A8G IN PRACTICE
CASE 13 History A 79-year-old female has just been adm..illed to the general surgical ward to have a large bowel tumour surgW:ally remon!d. The tumour
discovered at colonoscopy after she presented 10 her. doctor with a six month history of rectal bleeding. \v... s
On admission she appearroto bese...erely short of breath and extremely tired. Further questiomng reveals thai her rectal blood losS has been no greater than usual. ---------~ Pul$e 100 betti/min
alood
pre~$l,Ire
Re$piro/Qry rate
100/80 nwnHg 24 breoiN./m,n
S002% (on air)
100%
--
-
I
J
THE A8G IN Pl<:ACTtCE
Arterial blood gas 06/06/2005 Ullit no 94J778 fD: Ethyl Swoiluon
On air
Normol H' pH
32.3 nlToI/l 7_49
Pco,
3_31 kPo
BiCarb
25 mmHg 11.9.Pa 89 mmHg 22rrmol/l.
8' 5"" ,",Ia~
K
No CI ,Co· Hb
Gluco5e
-2 mmoI/l 99.8% 1
J.B mmol/l 138 mmol/l 96 mmol!l 1.17 ll'I ",oIA 6.8 g/dI. 3.9 mmol/l
135-45} 17·J5-7.A5) (4.7-6.0) {35-451 l" 10.6) I" 801 122-2B} 1-2-+2) (,. 98%) (0.4-1.5)
(35-51 fl35-J.45)
195-1051 (1-1251 (13-181 (3.5-5,51
Questions
1. a) Describe her gas exchange? b) Describe her acid-base stale':> 2. What is the most likely cause of her breathlessness? 3. What would be the most effe...·th,e way of improving ~ delivery 10 her li.%u('S?
83
THE ABC IN PRACTICE
CASE 14 History A 78-year-old woman is admilled to the emergency department wit sudden-onset, g('fll."ralised, severe. abdominal pain. She describes the pain as being colicky with no particular f
On examination, the patient is h..x'lTlod~"l1"mically stable with war well-perfused peripheries. Despite severe abdominal discomfort, abdominill examination is relatively unremarkable: the abdomen 15 soft in illi (our quadrants and is only tender on deep palpation. No hernias or aneurysms aT\' palp.1ble i100 l'('Ctal exammation is unremarkable. An abdominal and erect chest X-ray are taken and found to be
normal.
During the course of the examination her dinical condition deteriorates and she is mon"j to the resuscitation area.
••
THE ABG IN PRACTICE
Arrreriol blood gas 10/08/2006 Umlno.:n34211
On 10 L O 2 by mask
10: Su-. Ultik H' pH Pc",
8icarb
8E
5"" Lodo1e K No
CI iCo'
Hb
Gloco.e
525nmal/l 7,28 4.39 kPo 33 mmHg 28.6 Jd'o 215 mmHg 16.2 mmoI/l -10.4 mrnol/l 99.8%
Nonnal (3,S.....4S1
fl·35-7.4S} IA.7-6,Oj
(35-451 I> 10.6}
I> 801 (22-2.) {-2-+2) (> 98%1
3.2
10.4-1.5)
4.6 mmol/l 135 mmol/l 96 mmol/l. 1.16 mmol/l 12 g/dl 3.8 mmol/l
13.5-5) 1135-1451 [95-I05} {I-US] 113-181 (3.5-.5.51
Questions
1. a) Describe her gas exchange. b' Describe her acid-base status. 2. What is the most likely diagnosis?
IS
THE ABG IN PRACTICE
CASE lS History A 35-year-old woman with type 1 diabetes is brought to the emergency department by ambulance after being found in her hou severely unwell. Following a discussion with her parlner it emerg(>'S ~ has not been eating for the past fe-.\' days due to a vomiting illness and, as a precaution. has also been omitting her insulin.
Examination On examination, she appears drowsy and peripherally shut-down, with very dry mucous membranes. Her breath smells of acetone an her respirations are deep and sighing.
I M" Blood
preswf.
Resplfatory fOIe
! ~ (blood glLlOO5eI
130 beob/min 100/60 mmHg 26 breaths/min :>- 25 mmol/l
Ph}"Sical examination of her chest and ..bdomen is unremarkable.
••
THE ABG IN PRACTICE On 10 L ~ by mask
Arterial blood 9Q$
27/02/2005 Unit 1'10•. 7773S566 10: IlJa Tonner
Normal
H' pH
-.
Bicarb
BE SA>,
""K No C! jCo~
Hb Gi... :;o~
88.9 nmol/L 7.05 I 5 !.Po II ml'llHg 28.4 kYo 187 mmHg 6.0 mmoI/l -25.2 mmoI/l
99."" I 4.6 mmol/l 141 ",moI/l 96 mmol/l 1.25 mrno!/l 12 g/dl 35 mmoI/l
135-451 {7.35--7 ....S} (4.7-6·01
135-451 I> 10.6) I> aO) 12.4-30J 1-2_2} (> 98%1 (0.4-1.5)
IU-51 (135-14.51 (9S-105J
{I-1.25J 113-18)
(3.5-5.51
Questions 1. a) Describe her gas exchange. bl Describe her acid-b."\Se status.
2. Calculate the anion gap
3. What is the m05t likely diagnosis?
•
.7
THE ABG IN PRACTICE CASE 16
History A 37-year-old well-known local vagrant is brought into the department unconscious. He was fowld near a bottle of vodka and half-empt)' bottle of what appears to be methanol. It is unclear if he has drunk any of the contents. Exomination
The patient is unkempt with a significantly reduced conscious level (Glasgow coma scale score:= 9). There are no apparent focal neurological abnormalities.
II
THE ABG IN PRACTICE Arteriol blood gos
On air
10/07/2006 Unit rlO,: 35477899 ID: Gary Sooness Normol
W
63.3 rlmol/l
(35~451
pH
(7.35~7.45)
Bicorb
7,20 3.3 kPa 25 mmHg 12.8 U'a 96 mmHg 9.5 mmol/l
BE
-16,2 mmol/l
Sit>,
97.8% 1.3 4.5 mmotjL 136 mmol/l
Pco,
lactate
K No
(I
iCo' Hb Glocose
99 mmol/l \.1 mmol/L 13.5 g/dl
3.8 mmol/l
{4.7---6.0] (35--4':::')
(> 10.6) (> 80) (22-28) (-2...-+2) (> 98%) (0,4..-1.5) (3,5-51 (135-145) (95-1051 fl-L25] 113-18) P.5-5·51
Questions 1. a) Describe his gas exchange.
b) Describe his ilcid-base status.
2 What is the anion gap? 3. Is the acid-base status consistent with methanol ingestion?
.9
THE ABC IN PRACTlCE
CASE 17 History A 52-year-old man is being investigated on the urology ward fOT recurrent renal stones. He also complains of mild fatigue and lethargy. There is no history of gastrointestinal disturbance and he is not on any regular medi1:ations. Examination The patient is weU and clinical examination reveals no abnormalities
As part of the investigations an ABC is obtained.
90
THE ABG IN PRACTICE ArrcrioJ blood go~ 23/04/2006 U",JI no.: 27634943 10: Roger Porry
On air
Nonnol
H'
pH Ito,
""
S
"""'" K
No CI
iCo· Hb G1UCOle
43 nmolIL 737 4.2 kPo 31.5 mmHg 13.2 kf'o 990 mmHg 18 mmol/l -7 mmol/L 99'% I 3.0 mmol/t 137 mmol/l 109 mmol/l 1.0 mmol/t 13.0 g/dl 4 mmol/l
13~
(735-7..45) (47-6.0) {35-45} I> 1O.6} 1>801 122-281 1-2-+21 I> 98%1
10..4-1.51 (3.5-51 [135-145) [95-105)
(1-1.251 1l1.5-15.5)
13.5-551
Questions
1. a) Descnbe his gas exchange. b) Describe his add--base status. 2. CakWate the anion gap_ 3. What is the likeliest diagnosis?
.,
THE ABG IN fl:ACTlCE
CASE 18 History An 18-year-old is admilled to the toxicology unit ha\ing taken a large overd06e of an unknown substarKe 5 hours earlier. She complains of nausea and a high-pitched noise in her ears.
Examination On examination, she is mildly confused. Her respirations are increased In both fale
""~ Rllspiratory rOM Bbxj pl"IIUU'1I
'Tenperatl;,.
0,"
92
100 beals/min 26 moottn/min 132/100 mmHg 37.6'"<:
....
THE ABG IN PRACTICE Arteriol blood gos
On air
10/09/2006 Unit no,: 2763.49.43 10: libby Forqh\lor No<~1
".
38.8 nmoljl
(35-45)
pH
7 ..41 3.01 kPo
(7.35-7..45)
14.7-6.0)
22.6 mmHg
135-451
14.1 kPo
I> 10.61 (> 801
Pto,
,."
97.5 mmHg 176 mnd/l -8.3 mmolJl 99%
!octo..
1..
K
3.6 mmol/l 140 mmol/l
Bioo
No
CI ICO·
Hb Gfuco~
99 mmoI/l 1.2 mmol/l 13.0 s/dl 5 mmoI/l
122-281 l-2-+21 I> 98%) (0........ 1.5)
13·5-51 (I35-1.451 f9S-1051 11-1.251 111.5-15.5)
13.5-5..51
Questions 1. a) Describe her gas exchange.
b) Describe her add---base status. 2. Cak-ulate the anion gap.
3. What substance is she most likely to have taken?
THE ABG IN PRACTICE
CASE 19
History A 24 year old man is brought to hospital by his family after becoming unwell at home. They I't'port thai he has no history of significant medical illness but has recmtly been undergoing tests for progressive fatigue and weight loss. Over the last few days he has become illcrcasingly weak and lethargic and has also complained of muscle cramps. They became alarmed today when he appeared drowsy and disori(,·nted.
Examination TIle patient appears listless and confused. He has cool peripheries and pvor C3pillary refill. He is afebrile and lhere is no rash, Iymphadenop
Pul~
Respiratory role
8Iood
preswre
Temperature ~
.4
120 beats/min 25 breot!ls/min 75/55 mmHg 36SC 2_.•
-,
THE A8G IN PRACTICE Arterial blood gos 10/08/2005 Unit no.: 456986793 10: Fh..r\l~ Wainwright
On_
"",~I
H" pH
""" Po,
a-b
"
So,
'octo.
K No
CI
""Hb
G1uco$e
48 nmol/l 7.32 3.) "Po 24.8 mmHg 13 "Pa 97 mmHg 13.4 IM'OI/l. -1).9ml'l'lo(fl
""" 3
5.6 mmol/l 125 mmot/l 101 rrnmi/l
(35-45(
(7.35-7.45} (4.7-6.0) (35-451
I> 10.6)
I> 80) (22-281 l-2-2J (> 98%1 {O.4-1.5} 13.5-51 (135-1451
195-1OS(
1.2 mmotjt
(1-1251
13.0 sId!.
11~181
2.5 mmollt
(3.5-5.51
Questions 1. a) Describe his gas exchange.
b) Describe his acid-base state? 2. Are there any other abnormalities on th(' ABC? 3. What specific t::reabnent does this patienl require?
.s
THE A8G IN PRACTICE
CASE 20
History An 87-year-old man has been found collapsed in the ward toilet. Cardiopulmonary resusdtation was commenced promptly, as no pulse or respiratory effort was detected, and has now been in progress for 12 minutes. He has a past medical history of ischaemic heart disease. dementia and chronic renal failue.?
Examination
The patient has a Glasgow coma scale score of 3 and appears pale and mottled. The cardiac monitor reveals an agonal rhythm as shown below. No pulses are palpable and there is no respiratory effort. He is currently being ventilated by a bag and mask on 15 L/min of~.
••
THE A8G IN PRAGICE
On 15 L O 2 by bag mask
ArJerial blood 90$
10/02/2004 Unit no.: .42333993 ID: [)ovid King W pH
reo, Po, Bicarb
BE So, lackJte
,
No C1
Co· Hb GllICQSe
Normal (35-451 (7.35-7.45) (.4.7-6.0) (35-451 (> 10,6) (> 80l (22-281
160 nmol/l
6.' 4.8 kPa 36 mmHg 32 \;Pa 240 mmHg 3.8 mmol/l -20 mmol/l 100% 9 4.5 mmol/l 136 mmol/l 96 mmol/l 1.1 mmol/l 14.0g/dl 4 mmoJ/t
1-2-~2)
I> 98%1 {0.4-1.5) (3.5-5) 1135-1451 {95-1 051 11-1.25) 113.5-18.5) (3.5-55)
Questions
1. a) Describe his gas exchange. b) Describe his acid-base status.
2. 'vVha1 is his prognosis?
97
THE ABG IN PRACTICE CASE 21
History A 59-year-old male with a history' of alcohol excess presents to the emergeocy departmc:nt with a 3-day history of 5e\'e"re uppc:r abdominal pain. He now also reels \'ery' breathless. He ddmits to drinking up to 100 units of alcohol per week for the past few wee
Examination appears very unwell. He is tachycardic (120 beats/min) and hypotensive (75/60 mmHg). Th is marked epigastric tenderness.
Th{' p.."\tient is in f'vi(jpnt
(ji<;tn..,.;.~ and
A venous blood test taken on admission reveals a grossly elevated amylase (1890 U/ml) and C-reactJ.ve protein (274 mg/L). A chest X-rayon admission is shown in Figure 34.
THE ABG IN PRACTICE Arle1iol blood 905
On 15 L O 2 by mask with
10/08/2005 Unil no.: 45679393 10 Dofli~l Corter
reservoir bog
NOImol H"
pH Pro,
Bi<mb
"S""
loclo~
•
No CI
47 """"It 7.33 3.24lPo 24.3 mmHg 10.8 kPo 81 mrnHg 14.9 n-wnoI/t -11.8 mmol/l. 99% 3. 1 3.6 mmol/L 1.4\ mmollL
135-"51 (7.35-7.AS}
14.7-6·01 (35--451
I> 10.61 I> 80) 122-28) {-2-+21 (> 98%) 10.4.-1.5) 13.5-5)
96 mmol/l.
(135-1451
Hb
0.89 mmol/L 12.0g/dl
(95-1051 (1-1.251 p3.5-\8.5)
Goc~
16 mmol/L
(3.5-5 5)
i(o'
Questions
1. a} Describe his gas exchange. b) Describe his acid-base St.,tU5. 2. \\'hat is his approximate FiO:z?
3. \Vhat is the most likely diagnosis?
..
THE ABG IN PRACOCE
CASE 22
Hi5tory A 35-year-old woman on a g}"naceoJogy ward den~lops a severe ...omiting illness one day after elective steriJisation. She continues to vomit profusely for a further 3 days. Examination of her fluid balance chari re\'eals that she is failing to kc:..oep up with her fluid losses but has not been prescribed intra"enous fluids.
Exomination The patient appearsdehydraled mUCO\lS membranes.
WIth
reduced skin turgor and dry
Abdominal examination is unremarkable. p",~
100 beab/m,n
Re~pirotory rale
10 brectfu/mirl 160/100 mmHg
Blood preswre , lemperOTure
I 0,"
100
.."
36_6~C
THE ABG IN PRACTICE
Arterial blood gas 23/08/2006 Unill1o.: 27634943 ID: Jenny A1egranza H'
36 nmol/l
pH
7.44
/to,
(I iCo'
6.4 kPo 48 mmHg 11.1 kPo 83 mmHg 32 mmol/l +4 mmol/l 96% I 3.0 mmal/l 133 mmol/l 91 mmol/l 1.0 mmoJ/l
Hb
11.°9/dl
Glocose
5 mmol/l
1'>, Bicarb BE 51'>, lactate K
No
On air
Normal 135--451 17.35-7.45} 14.7--6.0)
135-451
I>
10.6)
I> BO)
122-281 l-2-+2} I> 98%1 10..4--1.5) p.5-5} 1135-1451 195-105) 11-1.25) 111.5-15.5) 13.5-5.5)
Questions 1. a) Describe her gas exchange.
b) Describe her acid-base status. 2. \Vhat electrolyte abnormalities are present?
3. \Vhat lreatment will correct the acid-base
101
THE ABC IN PRACTICE
CASE 23 History An 8-w('('k-old child is brought to the emergency department, with
weight loS/> and projectile vomiting. The paIT'nts report that he had an uncomplicated delivery with no postpartum complications. He initially fed well, appeared to be thriving and gave no cause for concern but has deteriorated markedly over the past 2 weeks, vomiting all of his meals and now losing weight.
Examination The child is agitated, crying and is malnourished. His mucous membranes are dry and oncxamination of his abdomen a small m is found in the epigastrium.
102
THE ABG IN PRACTICE capillary blood g05 05/10/2005 U"iIIlO.: 28734943 ID: Richard Barter W pH
Pw,
BiUlrb
BE
"'"
laerate
K No (I iCo'
Hb
29 Ilmol/l 7.54 6.1 1&'0 .45.8 mmHg 11.2 ~Po 80 mmHg 37.5 mmo!/l +14 mmol/l
99% 1 2.5 mmol/l 135 mmol/l 86 mmo!/l 1 mmol/l 1B,O gJdl
On air
Normol {35-451 17.35-7..451 14.7-6.0} tJ5-451 (> 10.6) (> BOl (22-2BI (-2-+21. (> 98%1 (0.4-1.5)
13·5-51 (135-1451 (95-105) (1-1.251 (13.5-18.5)
Quesfions
1. a) Describe his gas exchange. b) Describe his acid-basc status. 2. Considering the pH and He03: is the PaC~ IUgher or lower than you would expect? 3. What is the underlying diagnosis?
'03
THE ABG IN PRACTICE
CASE 24
History A 36-year-old pregnant woman 011 the maternity ward complains feeling short of breath. She ha~ no other symptoms and no relevant past medical history.
Examination On examination, the patient is heavily pregnant but appears otherwise well. Examination of her chest re....eals no abnormalities.
Pube Respiratory role Blood prenure
110 beots/mirl 20 breaths/min 112/100 mmHg
Temperature
36.6"(:
0,%
99%
An AI3G is performed.
104
THE ABG IN
AfMriol b600d gas 18/08/2005 l,klill'lO.: 27634943 10 julie Dcnoldson H"
pH Ito, Po, Bicarb
"_Ie , SIb,
No
C; iCa' tf> GI.
35 nmol,/l 7.45 4.9 kPo 35 mmHg 4.7 Id'a 35 mmHg
24.0 mmol/L 2 mmoll\. 74% I 3.6 mmol/l 1381nf1'1oOl/l. 1Q.4 mmol/l. I. U mmol/1 13.0 g/dl 5 mmol/l
P~ACTICE
On air
Neomal (3)-<5) (7.35-7.45) 14.7-6.01 (35-.451 I> 10.6)
135-t5)
12WOl 1-2-...2) I> 98%l 10·4-1.51 13.5-5) (135-1451 (95-105J 1I-1.2S) (11.5-15.5j (3.5-5.5)
Questions 1. a) Describe her gas exchange. b) Describe her itdd-base status. 2......'hat is the mosllikely ell.ptanation lor the low
P~?
'0'
THE ABC IN PRACTICE
CASE 25
History A 55-year-old woman on till> orthopaedic ward complains of suddcn-onset breathlessness and pain on the left-hand. side of her chest. She underwent elccti\·e knee replacement surgery 4 days previously and has!x.'('3\ immobile in bed since the operation. She is otherwise well with no rele...· ant past medical history. Examination
The patient appears well but slightly short of breath. Other than mild tachycardia and tachypnoea, examination of the cardiovascul and respiratory systems yields no positi....e findings and there is no clinical evidence of deep-vein thrombosis. A chest X-ray reveals no abnormalities and an electrocardiogram shows only sinus tachycardia
""~ Re1piratofy r
0,%
'06
98 beets/"'in 20 l:lI'eoh/llIifl 160/IOOrnmHg 36.6'"<: 99%
THE ABG IN PRACTICE ",terial blood go~ 10/08/2006 Unit no.: 27634943 ID: jill Arcl1erson
On air
Normol W pH
Pco,
fb, Bicorb BE SPC, lactate K No
(1
iCo' Hb GllJco~e
36 nmol/l 7.43 A,9!o:Po
37 mmHg 12.1 kPo 91 mmHg 25.8 mmol!l -1.8 mmol/l 99% 1 3.8 mmol/l 136 mmol/l
99 mmol/l 1.2 mmol/L 10.09/dL 5 mmol/L
(35-451 (7.35-7.45) (4.7-0.0)
135-451 (> 10.61 (> 80)
(22-28} (-2-+2) (> 98%} (0.4-1.5) (3.5-5) {135-1A5) 195-105) 11-1.251 111.5-15.5) (3.5-5.5)
Questions
1. a) Describe her gas exchange. b) Describt'o her acid-base status.
2. What is the A-a gradient? J. Does she require <my further investigation?
'07
ANSWERS CASE 1 1. 2. 3.
0)
Type I respirotory impoinMIIt {ll'lOlhorotel
bl
Uncompensated ,e$plrotory aftlC:~osis
Ye5 Ves
I his patient has modE't'atc Iype 1 respiratory impairmf"l"lt Hyperv~hlation is an appropriate response to ~ hypoxacmia and seno;ation of dyspnoea and has resulted in a. mild alkalaemia (remember lhut metabolic compensation does not occur in rESponse to QCute n..~piratory acid--base disturbance).
1nc lUJT@('t manal';cment for his rondition is supplt>lJlelltal oxyg~ to COtTfft the hypoxaemia and appruprialP antibiotic; to ~at the infection. In a patient sIKh as tlus, with moderate hypoxaemia and no vt'fltilalory impairment, monitoring by pulse oximetry is more appropriolte than n>peated ABG sampling. lru.lications for further ABC analysis would include f'igns of exhaustion or hypen:apnia (p. 23) or a further :.igmficant ~ in ~
109
ANSWE~S
CASE 2 1. 2.
01
QfOflic I)'pe 2 respirolofy impoirmenl
b)
Compell$Oled
f~irolofy ocidosh
Chrooic type 2 respirorory irrpoirment due 10 morbid obesity
At first glance it may be difficult to determine whether this ABC rcp~ts respiratory acidosis with metabolic compensation or metabolic alkalosis with respiratory compensation, as both give a high HC03 and high PtiCOz. The best due is the pH (or H') which, ahhough just within the normal range, is on the brink of acidaemia. ThIS would represent overcompensation for an alkalosis and. therefore, sugg~ an acidosis as the primary abnormality (overcompensation does not occur). The mildJ}' impaired o)()'genation is consistent with the degn.-e of h}l>o'H,:ntilation. 1be likeliest caU5e of chronic type 2 respiratory impairment in this case is se\"ere obesity. Around 20% of individuals with a body mass index greater than 40 have chronic hypercapnia from restricted ventilation (pickwickian syndrome).
110
ANSWERS
CASE 3 1.
01
Mild Iype I fespirotory impairment (wirfl morhd hyperventilation)
hi 2
Uncompensated resplfclory olkalosis Putmonory embolism
This poltient is a young. fit. non-smoker with no history of lung problems but, despite hyperventilating (low PacOV. has a P~ below the normal range. indicating impaired oxygenn.tion. Given the recent long-haul flight and absence of clinical and X-ray abnormalities. the most likely cause of her breathlessness and impaired oxygenation is pulmonary embolism and she must be investigated accordingly. This is oneaf the clinical situations wheTe ukulation of the .\-a gradient can be helpful (more so when the P~ is just .....' ithin the normal range). As shown below, it is signiflcantly elevated, indicating the presence of VIQ mismatch.
I
A-<J grodient - PA~ - Po~ PAD2 _ (0.2/ l< 93.81- (3.9 l< 1.2) _19.7_4.7 _ 15/t;Po A-o grodienl_ 15 - 10 3
.4.7 ;aorm <2.6 kPo) OR
PA02_{O.21 l<713J-/29.3x 12) - 150- 35 _ 115 A-o gfodieot • 115 -
n
• 38 mmHg (norm <20 mmHg)
---------'----'------'
.
,
ANSWERS
CASE 4 I.
2 3.
aJ
Acute type 2 re1plrafory fodure
b]
Uncompensoled rllspirotOf)' ocidosis
Opioid toxicity Op,otd ontogonisl
Opioids haw a depressant effecl upon respiration and may lead 10 acute \;t'I1tilatory failure (type 2 respiratory failure). This elderly man has received a large amount of morprune in a short period of time and exhibits pinpoint pupils. malting opiok! toxicity by far the likeliest cause of his severe ventihllory failure. Since metabolic rompcnsation takes several days to occur, the acute respiratory acidosis has produced a SC"t'Il! acidaelJlia. His P~ though within the normal range, is lower than expected f~ a pilllt!'ni breathing 28~o Oz, and more or less consL'itent with this degree of hypoventilation.
In addition to basic life support meilsures, he should be adminislered an opioid ant'1.gonist (e.g. naloxone), to re\'erse the respiratory dcp~ion, then closely monitocro 10 ensure sustained impro~·em~nt.
112
ANSWERS
CASE 5 Type I re$pira:OfY Imparmen1lmoderalel
,
:::I)
2
bl Normal Yes
a<;i~tl(He
balooce
l1'e above ABC IS helpfulln guiding the correct t~atment for this pol'cnt with acu~ exacerbation of chronic obstructive pulmoRdry dl~~;a5€'. Patients wilh this condition are often prescribed inadequate ox\gen for feM of suppressing hypoxic drive (and thcJ:eby depressing ventilation), but this is onJy an issue in patients with ..-'IM'I!.:: /yF1I/' 1 respiratory failure (indicated by t PlICO:!. and t HCD,). Thl., patll~nt has typr 1 resptratory impairment and will not rely on h) p'lxic drive. Although this patient is likely to ha....e a chronically low 1'>1Q:, the 3(1.)t", deterioration in his symptoms and exercise tolerance suggests a rurther n.~t decline from his nonnal baselinf-. lmportanlly, l"·en a small drop in P~ around this }('vel (steep part ofOr-Hb curve) may C
ANSWERS
CASE 6 1.
2.
01
Acute Iype 2 respirotory impoirmet'll
bI
Uncompenwted respirokJfy ClC~1
Not
This patient with an acute exacerbation 01 chronic obstructive puJrnonary disease has been struggling to overcome severe ob6truction to airflow over a period of hours to da~"S and is now exhausted from the increased work of breathing. As a result, his alveolar ventilation is declining. leading to acute type 2 respiratory failure. Th..is JTh1.y complicate type 1 respiratory failure from any cause. not just from chrunic obstructive pulmonary disease. The rising p~ is. therefore, 'lOt due to diminished hypoxic drive and removing his oxygen willl10t correct it. Indeed his F~ should probably be incT('~ (in addition to other treatment) as he remai significantly hypoxaemic. Remember" thai. with acute respiratory acidos.is, there is no time for metabolic compensation to develop and a dangerous acidaemi.l. de"'elops rapidly. Adequate \"{~ntilation must be restored, as a matter of urgency. to COrrecl the PIlCOz. Possible measures. in this case, include a respiratory stimulant (e.g. doxapram) or, preferably. non-invasive ventilation. If these fail, mtuhalion and mechanical ventilation may be required, If considered appropriate.
ANSWERS CASE 7 01 bJ
2
Type 1 r~i,oklry impoirmef'll M Id re~rotory ollolosis bo onced by mild me'Oboi c ocidc~'s p,kely two p"mory processesl Aspiration pnevmo'l 0
J
Se¥efe
nu.. patient has mild to moderate hypoxaemiJ despite recei\'ing a hIgh Fi02 and therefore has severe impairment of oxygenation The ,;hf;htly low PilCo.: indicates that ventilation i" adt'quate, so this l~ II'PC I n..~piratory impairment. The probable expl,mation for the mild Illct3boli.::: acidosis is lactic acidosis resulting from tissue hypoxi", The
lustoI)~
examination findings and chest X.ray all suggcst a Ji.lgOO5is of aspiration pneumonia.
nus patient is se\"erely unwell and any further decline in her P0O:: could be catastropttic (on the steep part of the OrHb .aturation curve). Her 0:l therapy should be adjusted a.., necessary 10 mamtain ht.-r 5Doz abo\"c~.. and she should be managed on iI htgh--d.epcndency unit with dose monitoring for- signs of deterioration.
ANSWERS
CA5£ 8 1.
'"
bt 2. 3.
Chronic type 2 re~rotocy irnpaiflMnt Compensated respirolory ocidosis
PO, PC~
and HC03
Acute exacerbation of chronic obstructive pulmonary disease is a common medlcal emergency and this case illustratt's key principles. ], The f p~ shows the patient has type 2 respiratory impairment (ventilatory impairment). 2. The f HeOl tells u.s it is chronic type 2 impairment (since metabolic compensation I~ time to develop), 3 Even in chrt'lnic type 2 impairment, a further DiCute rise in Poco: would lead to an acidaemia. The pH hen? is normal, so Ihe PtKOz h.1S not changed appreciably in the last few days (i.e. there is no acute-on-:z.lS likely to ha\'e dropped, leading to the increased breathlessness and marked decline in eJt"erc1se capacity: bel a PIJ~ of 8 kPa (60 mmHg) ev<.-'ll small falls may cause a rna' decline in $QO;z (steep part of the cun'e). 5, As this patient has chrortic hypercapnia, he may rely on hypoxic drive as a stimulus to ventilatlon, 6. The goal is to ensure adequate oxygL'Oation (we must not ignore his hypoxaemia) without depressing ventilatory drive
"6
ANSWERS
CASE 9 1. 2
01
AcullH)l'Khronic type 2 re5pirotory imfXIirm6rlt b) Partially compensated respiratory acidosis Excessive supplemental O 2
Care must ~ taken when prescribing supplemental Oz to patients with chronic type 2 respiratory failure. The aim is to reverse ,my recent worsening ofhypoxaemia and allow adequate tissue oxygenation, without depressing ventilatory drive through an excessive rise in PaO:l. Most authorities recommend controlled O 2 therapy using a fixedconcentration mask at an initial concentration of 24-28%. The response to therapy must be dosely monitored with frequent clinical assessment and repeated ABC measurement. Note that pulse oximetry is /lot an adequate substitute for ABC in these circumstances as knowledge of the Sao:z alone does not permit assessment of ventilatory adequacy. The latter point is well illustrated in this case as the-patient now h,lS life-threatening acute-on-ehronic respiratory failure despite a normal ~. The rising Pac~ must be rapidly checked and reversed through improving ventilation. Potential strategies include reducing inspired 0, cnncentration, respiratory stimulants or assisted ventilation.
117
ANSWERS
CASE 10 a)
Miki type I re$pimtary impairment
hi 2. J.
Normal The P~ (high end aI ncl'TIal ronge)
Life.lhreatening ahad
This patient has sen~ral features of a ~"ere asthma attack but it is the high-normal PilCO} that is the most worrying aspect of her presentation and makes it a life-threatening attack. Patients with acute ex.ocerbalions of asthma should have a low P~ on account of the increased respiratory rate and effort (t alveolar \·entilation). A level)o 5 kPa (37.5 mmHg) suggests the patient is strugghng to overcome the obstruction to airflow and. perhaps. beginning to tire from the effort of breathing. Consequently. her Paco:: signals a life-threatening attack. The intensh·e care unit should be informed irnmediaklr of any patient with acute se,·ere asthma and Iik--threatening features. P<1tients must receh"e intensive tre<1tment and monitoring, including repeated ABC me,lsurcmomls to assess n.--sponse and identiIy the nCl'od for intubation.
n.
ANSWERS
CASE 11
2.
01 Hyper\/'llfllilorioo lp"moryl bl U~peruoted respiratory oll:alosis Low ionized coki\,lll'l
3
Psychogenic hypervenlilotioo
I
This is a classic clinical picture of psychogenic hyperventilation. The ABC is entirely in keeping with this diagnosis as it reveals e\-idence of hyperventilation (low PIl'O:>V with normaJ oxygenation (nannal pQO:z) and a normal A-a gradient. Note that the HC~ (and BEl is norma.l as there has been insufficient hme for metabolic compensation to OCXUf. Consequently the rt'duction in FiKOz has caused a marked alkalaemia. Anothet" point to noll' is that the concentJation of ionized calcium m the blood is afft>eled by the pH of the specimen. since H' ions compete with cakium for binding sites on albumin and other proteins. Therefore, as the number of H+ ions drops (alkalaemia), more caldum is bound to albumin, causing serum Ionized calcium levels to fall. This is the most likely cause of the numbness and tingling. Care must be taken in ruling out other cardiovascular and respiratory pathologies before ascribing s)mptoms of chest pain and shortness of breath to psychogenic hyperventilation.
119
ANSWERS
CASE 12 I.
0)
Normol- bul w.ere hypoxaelftio
bl
2.
J.
COlTIpeoSOled metobolic ocidosil COfbon monoxide poisoning Soc,
Carbon monoxide (CO) poisoning commonly presents with nausea, vomiting. headache and ronfU5ion. CO saturations correlate poorly with symptoms but levels above 5O'r~ may cause cardiac arrest and seiz.ures.
Why is the patient hypoxaemic? CO binds to hacmogJobin (Hb) with 200 timts the affinity of ~ so is carried on the Hb molecule (as carboxyhaemoglobin) in preference to ~. As a consequence. the perrentagc of Hb saturated with Oz - the ~ - is maTIcOOI~' redl.K."'ed in CO poisoning. f",'cn when the p~ is very high. Since the overall Oz content of blood is determined by the Sooz and Hb concentration, ~ dclivcry to tissues is inadequate (tissue hypoxia), le."lding to lactic acidOl'iis.
Why does .5cJo, appear to be normal? Most pulse oximeter.> are unable to distinguish carboxyhac:moglobin from oxyhaemoglobin, so fail to reflect the truc ~ in CO poisoning In ABC analysis. the ~ is not norrnaUy measured but simply COlIeu1atro from the measured PaOz. The Jailer parametcr is based only on free, unlxlUnd O 2 molccules SO is unaffected b}' the presence of CO.
120
ANSVVERS
CAS! 13 I.
01
HyperverJtilation _ 00 impairment of oxygenol;Of\ bul hypoxoemio
se<:ondary 10 onoemio (P.I 81
2. 3.
bl Uncompen~ted re$pirotory alkalosis Anoenio (Hoemoglobin 6.81 Reslolft hoemog!obin {Hbt <:onc:anllo!ion (I.g. blood transfusion, ilon
replocementl
This patil.'I1t appears to have severe anaemia, most likely due to iron d"fKicrlC)' from dmmic ra"tal bll"l'dlng. This should be rontirmed with a formallabocatory sample ('full blood counr). There is no impairml...'I1t of ventilation or O 2 transfer so Pae>. and SaO:! are normaL However the vast majority of O:l: in blood is carried by Hb, so the overall O:l: conlmt of her blood is low. Hyperventilation is a nonnal response to the sensation of breathlessness and increases the PaO:l: slightly" How€\u, in the above context, this has very little impact on blood O:! content as the available Hb molecules are already fully So'lturated: For the same reason supplemetal O:! would also fajl to improve O~ content significantly. Indeed the only effecli\"e stTatt:.-gy is to increase Hb concentration. This could be achieved rapidly. by a blood transfusion or more gradually by iron replacement.
.2.
ANSWERS
CASE 14
2.
oj Hypen!entilolion (secondary! bj Partially compenwled meklbotJe acidosis Mesenteric ischaemia
What does the ABC tell us here? There is acidaemia due loa severe metabolic acidosis and the elevaled lactate tells us it is a loctlc acidosis. Lactic acid is produced by tissues recch'ing an insufficient supply of O 2 bul oxygenation is normal (note the PtlOJ is appropriate for an F~ of -40%) and there are no clinical signs of shock (e.g. hypotension. cold peripheries) suggesting there is no generalised problem of ~ delivery to tissues. In fact, the source of lactic acid here is bowel. The patient has mesenteric ischaemia, in which blood suppl}' to the bowel wall is impaired due 10 occlusion of an artery by a thrombus or embolus. In the absenct" of an adequate blood suppl)~ bowel tissue becomes hypoxic and must rely on anaerobic mrlabolism (producing lactate as a byproduct).
Mesenteric ischaemia is a difficult diagnosis to make. as presenting symptoms, signs and routine im'estigations are all non-specific. diagnosis should be considered in patients with minimal abdOUll1\l1 examination findings despite severe pain. especially in the presence of a lactic acidosis.
122
ANSWERS
CASE 15 Hyperventilation (lecondory) bl Se-e rretobolic ocickW$ with partial compenKition AIltcn gop • 0.4 1 + 4.6) - (96 + 6) • 43.61nonnal. 10-12) oj
2. "::
Diaberic ketoacidosis
In dmbetic keto.1cidosis, severe insulin deficiency leads to hrpcrglycaemia and incrt'ased metabolism of fats. 'The bl't"akdown
(If fats produces ketone bodlCS - smaU organic acids - which provide lin alternati'l;e source of energy but can accumulate, leading to a rrofound metabolic acidosis. [t is the ketone bodies thai account for I~ raised anion gap. In this case the acidosis has overwhelmed not only the kidneys' ability to excrete an acid load but also respiratory compensatory mechanisms. 1herl' is therefore a severe and dangerous acidaemia despite near-maximal respiratory compensation. An equally important problem in diabetic ketoacidosis is the profound osmotic diuresis resulting from hyperglycaemia that leads to severe dehydration and electrol}'te loss.
ANSWERS
CASE 16 1.
01
2. 3.
bJ 32 Yel
Hyperventilation I~andary' Sevel8 metabolic ocidalil wi'" portial compenWlioo
Methanol ingestion can be fatal in doses as small as 30 mL Methanol is metabolised by the liver to produce formaldehyde and fonnic acid. Accumulation of formic add leads to a profoWld acidosUi with a raised anion gdp. It also ca~ ocular toxicit)' and may result in permanent blindness. Anion gap.: (Na /1361 + K 14.51)- (CII991 + H~ 19.51) "" 140.5-108.5 '" 32 Treatment for methanol poisoning is complex but often involves the adminislTation of ethanol, which inhibits the conversion of mcth..mol to its more toxic metabolites.
12.
ANSVVERS
CASE 17
2
3
0)
Slighl hypenoenllotion lsecondory)
bI
Compersated metabolic ocidosis
Aniongop -1137+3.0:1-(109+ lSI _ 138-125 • 13 (normoq RenolllJoolor ocidcsiJ (type II
The diffurentiaJ diagnosis of normal anion gap acidosis is rcL"ltively n.\rrow and, in the absence of diarrhoeal s}·mptom.<;, ~al tubular i1cdosis (RTA) is the likely cause. The history of renalstoncs and a...."ocialcd hypokalaemia also supports the diagnosis_ In type 1 RTA,lhe kidneys fail to to secrete H' ions into the unne in l''(\--h,mge for Na' ions. This leads to e>.:cessh·c loss of HCO:J in the urule, resultitlg in an acidOSIS. To maintain e1cclroneutrality, e>.:ua CI U.lnS are n..-taincd (so it is a "~rchJornt!mjc acidosis). Becau~ 0- is a m{'asured ralher than unmeasured anion, lhere IS no increase in the anton gap. Type 1 RTA is often complicated by renal stones as calcium lends to rrecipilate in alk."lline uriTll', fonning stones.
H}'pokalaemia results because "Ia· ions are exchanged for K· ions Ilt>l... ad of H· IOns.
.25
ANSWERS
CASf 18 1
01 b]
2. J.
Hypervenlilofion Compensated metabolic ocu:I01.i1. 0( metaboLe ocid01.is with concomitcmt re1.pFrolory oIkoklsis
27 Aspirin
5.llic~'late
poisoning may c.'luse both a pnmaI)' respiratory alkalosis (direct stimulation of the respirator), centre) and a primary metabar acidosis as saliC}'l.1te is an acid (it may also promote lactic acid formation). It is therefore not pos..o;;ible to confidently ascertain whether or not the hyperventilation is due to a primary cffe<:t of the aspirin or is a response to the metabolic acidosiS.
The diagnosis would be confinned in this instance by taking salicylatlt bels.
n.
ANSWERS
CASE 19 1.
oj
HyperventilatIon lsecondary}
Severe metabolic oddosi~ with portiol COO'IpenlOtion Hypo"IQIfOOMIO II No}. hyperl.oloemio I t Kj, hypoglycoemio I • glucose), ro sed b:lale ItllraYeOQUs l:Oflicoslero:ds hI
2 J.
This patient is likely to have adrenal insuffidency, a condition in which the adrenal glands fail to produce sufficient amounts of the hormone cortisol (and, in some cases, aldosterone). It often presents non-specifically with fatigue. malaise, anorexia and weight loss can be easily overlooked or misdiagnosed. Llck of adrenal honnones causes sail and water depletion and loss of vascular tone which, as in this case. may lead to dramatic circulatory collapse (acute adrenal crisis). The circulator), shock is the cause of the severe lactic aadosis. Patients also ha\'t" an inability to mobilise glucose ~r\res, h<'nce the hypoglycaemia, and characteristic electrolyte abnormalities (j Na: I K).
In addition to basic lire suppoc-t and fluid resuscitation. the main intervention likely to improve this patient's condition is the administration of intravenous hydrocortisone.
127
ANSWERS
CASE 20 1.
01 ~
2.
Normal (i.•. soliJocIory Yllflhlatioo and oxygenotion being ochieyed by bog orKl roosk) $eYere metabolic acidosis funcOlflJl8NOtedl
Prognosis
Very poor
1n i\ cardiac arrest case, the ABC has several uses. It allows one 10 dett'mline the adequacy of ventilation (iII this case provided manually by bag and mask), to identify the presence of hyperic:alaemia (o~ of the reversible causes of cardiac arrest) and may providE.' valuable progn~tic infonnatioo.
Hen', despite the evident success of bag and mask \'entilation in elinunating CO} and oxygenating blood, the patienl has a profound lactic acidosis secondary to over,o..-helming tissue hypoxia. Althaug tills may be multifactorial (e.g. the umkrlying disease process; hypoxacmia prior 10 bag and mask \'cntilation), the most importan call.<;£' is madequale: tissue pet'fusion due 10 loss of cardiac fWlction. This polticnt has a grave prognosis. 11le level of acidosi5 is unlikely to be compatible with life and, given his unfamurable rhythm (an olation is extremely unlikely.
128
ANSWEIlS
CASE 21
2.
Type I relpiratory impairmenl 1~V'8fel bl Severe fl'Ieklbolic acidosil with partial compen$Olion 0.6-0.8 f- p. 161
3
AcIOO par'lCfealllil
0)
The diagnosis of acute pancreatitis is based on the history. exam findings and increased serum amylase: the main use of the ABC is in :Jelping to assess illness severity. The severe lactic acidosis (high anion gap and incremled lactate) indicates m.uked tissue h)'J'Oxia. The main caUS(' is impaired blood supply to bssuesdue 10 circulatory collapse. This occurs as part of the systemic inflammatory response in pancreatitis and must be urgently corrected with aggressive fluid resuscitation:t. vasopressor agents. A POO:1 of 10.8 with an FiO:z of 0.6--0.8 implies severe respiratory imp.linnent, and together with chest X-ray appeara.nces, suggests the devclopment of acute respiratory distress syndrome (an Inflammatory lung condition). Although PaC>]. is currently adcquate, 1m:. p.ltient may require ventilatory support if he deteriorates further ('r tires from the increased work of breathing. Finally, the ABG also shows a high glucose and low calcium concentration - both of which are adverse prognostic fadors in acute pancreatitis. Th£ patient should be transferred immediately to a critical care ('TWlronment for intensive moniloring and supportive treatment.
129
ANSWERS
CAS022 I.
2.
3.
Mild type 2 re~1ofy impairment f<:ompen$OlOr)' rEt$ponse} Compon$llled metabolic alkalosis ElKtrolytes Hyp::*alaemio UIQ Hyponotroemio UNol Hypochb..-n.o ucq Treatment Auid and electrolyte repboement
0] b)
Vomiting causes loss of H+ ions in gastric juice. The normal response of the kidneys to loss of H+ ions is increased excretion of HCO,- to restore acid-base ba1aoc'e. So why dots this not happen? The reason is that persistent \'orruting also leads to fluid, Na, CI and K depletion. In these circumstances the o~'erriding goal of kidneys is
salt and water retention. Under the influence of a honnone called aldosterone, Na+ ions are retained at the expense of either K+ or H+ ions. Lf K+ ions were plentiful, H+ loss could be minimised, but K is also in short supply so both are lost (worsening both the alkalosis and hypokalaemia). 0- depletion also limits HCO)- excretion as there must be enough negatively charged ions in blood to balance the positi\'e!y charged ions (eJe:tlOneutrality). Thus, in this case, intravenous replarement of fluid and electrolytes (Na. CI, K) would allow the kidneys to excrete more HCOJ; ooile<:ling the alkalosis.
• 30
ANSWfllS
CASE 23 I, 2. 3,
0) Mild type 2 respiratory impoirmtmllcomper'lsolory responsef b) Nlelabolic alkalosis with partial compensation lower Pyloric stenosi~
Congenital pyloric stenosis is due to hypertrophy of the gastric outflow tract ~n the first 6 weeks of life. This obstructs flow between stomach and duodenum, leading to projectile vomiting and inability to absorb nutrients. As in the preceding cast', persistE!l1t vomiting has caused significant loss of H" ions, triggering a metabolic alkalosis. Again this lS maintained because the associated fluid, Na, and K depletion pre\'cnts the kidneys from increasing HCOJ excretion. However, in this case there is a severe alkalaemia for two reasons.
a
Firstly the metabolic alk.1.losis is more severe. This is mainly due to the greater duration of vomiting but also becau;;e obstruction between stomach and duodenum prevents HCOJ f~om the duodenum being lost in vomit. Secondly, there is minimall'E'Spiralory compensation. Given Ihe severe alkalaemia, one would predict a greater rise in PacC'1. lhis is probably explained by the child's distress, which provides an additional respiratory stimulus blunting the compensatory response (Le. a mild primary respiratory alkalosis).
.31
ANSWERS
CASE 24 1. 2
01 Appearance of .evere type I unpiratory impairment bl Normal acid.bose stalus Yer100s sample Explaootion
1be ABC result suggests severr. Life-threatening hypoxaemia but the patient appears dinicaUy well with onl)" mild symptoms and no signs of ma;or respiratory compromise. Moreover. there is a marked discrepancy betWl.'erl the ~ as measured by pulse oximetry (m,,) and thai calculated on the ABC (74%). By far the likeliest explanation .is that the sample was obtamed from a vern rather than an artery. A !Cpt-oat ABC should be performNl. It IS important to ensure that an ABG sample is obtained from an artery before using it to assess the P/lCIo;.. In addition to the points above. non-pulsatile flow and the need to draw back on the syringe at the time of sampling sugge'!it venous blood.
132
ANSWERS
CASE 25 I. 2.
3.
al b)
NOfl'l'ol gas exchonge No'mo1 ocid-bose stotus A-c gradient 19lPa!1S mmHg Ye$(mu!>I edude pul"onory embolismj
lnaflno~
This case is included to demonstrate the limitation of ABGs and the unportance of considering the clinical context, The ABC is entirely normal but the patient is at high risk of pulmonary embolism, given her ret:ent lower-limb orthopaedic surgery and subsequent immobility. Moreover, she has now developed sudd61.-onset pleuritic pain, breathlessness and tachycardia, unexplained by initial Investigations. While the finding of imp
PA0:2 - Pro,.
lfO 21.9381-14.9 x 1.2)}- 12.1 13.9 - 12.1 .. 1.8 U'a lnormal.< 2.6 U'aj
'« !
1[0.21 x713J-lJ7x 1.2}j-91 106 - 91 .. 15 mmHg (normal < 20 mmHg}
>-
'33
Index N.B. PlJg.. 'tI.mbt'rs in ilJlIi<' dmo1r muti'Tw/ ill Qur,;fwn 6' A"~'r !>'retlon
A-.'\ grndient 56, 106 7, 733 acid-b.1.SC balannp 26-35 blood pH, nonn"l 20 impo,t,mce 27 mainlt'lwnce 29 normaiJO. 31. 106-7, 1J1. 133 stalu§5S interp~l,llion54 ...-rulso pH add--b-41 acid-h....:.c disturbaT1('l."S 30-5 COffip<-'Itsated 32-3 mixPd34-5.41 uncomlX"ll::ioIted 30, J 1 .lC.d-base IlO<Jl<:lKraD'l 34,.\5 3Cldaemia. _'t>re 86-7, 12.1 aridity 26 ad.-lOSJS Tl a<:id ba5C' status 55 d..finibon 26,:lO su a/$.O l1Ietabolj.c a<;iJOI>llI; mixed addosis; nospiratory aodosis ilLut~ respirlllory distress "}'lldrome 98- 9, J29 ,..-tn.·n.d insuffictt:ncy 94-5, 127
alkalinity 26 alkalosis 21 acid-ba!\f' "',lIUll 55 definition 2", 1O St'I' ulsn metabolic illkaJosis; respiT.ltoryalkaloW; AIlPn lest. modified 11,47 ah·eol.1r-c<,pilJary mPmbrilrle" parli'll pn.'~ures alS ah'eolar g.t."ll"l.UiltiOn. simplifit.'
anatomy. n:spiratory 1 :!onion ~pp 37 in lIlPt"bobc acid05is 39, 5-1, 80-7. 90 1,125 severl' 88 9.98-9,721, 124,119 m salicyl.ul' poe>oning 91...J, 116 atll'nal blood gas (ABC) anal)'!'is common valu 'iO 1
..ontr...indicationo: 43 in cl,al;l_1S 48.-19 inli>Tp....--l.ltion 52-5 In practi~ (C'"~ 1itudi....) 59-108 m t:realmf'nl gUldaoct"1 monilonng 49 w;t'~ for 48-9 5« "/:;(1 urnplin& ....tf'l",al blood sampl~-M
INDEX aspiration pneumoniil 70-1, 115 aspirin (Sil.I~btl.') poisorung J.l,
92-J,126 assessment of illness S('\tl'rity, ABC in 48 l'~amples 49 iBthm..ll,life.-threat~ng 76--7, WI atmosphenc pres.o;ure 5
base excess (51::) allSeiSment of illness se..'erity 48 commOil ,"alul"S 51 in metabolic acidom 36 bicaroonate (~) actu,d (plasma) (HCo,.act) 51 ~tandard (HCO)o'>t)51 bkarlxmiltc (HCO:J) balance In add-base balance 28, 29 tr'I chronic hypen:apnia 22 ind~is4B
in hypt'l"'Cl\lilatiofl24 in metabolic ilcidosis 36 In mebbohc .llkalosis 38 in respiIilIOry impairmt'Tlt 23, 25 blood samples handling 46 venous ll$ arterial blood 44, 101--5, J3l buffers 26
caldum concentration, plasmil IOf\ised (iCa) common values 51 low 78-9, 98-9,119, 129 carbon dw,xidc (CO~ effect of shunting 15 elimination 6 and alveolar ventilation 13 in pulmorulry gas ~ 4
"6
carbotl dio~de partial prt'Slrurt' (PCOz)5 in air 5 in arterial blood (POlWJ.l 5 common values 50 III diagnosis 48 IDCrcased 6 Ul metabohc alkalosis 38 in mixed acidoW 41 r'l'a'ptocs 6, 7 in respiratOf)' addosi~!
alkalosis 40 in re6piratory irnpairmcnl20. 22 and VIQ mismatch 14
common values 50 and H' ;on remo.:al2B in rt'5J>iraloX)' iJnpainnerlt 23, 25 carboft monoxide poisoning 80-1, 110 cardiac arrest %-7, 128 chloride (CI") ions in met.moltc alkalosis 38 pIa5ma Jeoveis. oommon values for: 31 chronic ob!;tructive pulmonary dISeaSe 66-9, 72-5, lJ3-14, rt6-17 CI ion~ sef' chloride iOllS compensation '19,32-3 full ~ partial 32, 33 In metabolic alkalosis 38
(en
predicting 34 respiratory tIS metabolic 54 COIltraindicat;om to ABC anal}"sis43
diabetes rnelhluis
type 1 86-7. 113 type 2 60-1, J 10 diabettc ketoacidosis 32. 86-7, J2J diagnosis. ABC in 46, 49 disordeP.l of l\Cid-base balance 36-t 1 disorders of g3S exchange 18--25 dJuretic drugs 38
INDl'X
hydrogen ions (H') e1ec:trol)'be rq>laCl'.ment therapy IJl) eodotracheal intubation 16, 17 exh.a~
gas
composition during respiration
cycle 13 effec:t of shunting 15
fan" masks, oxygen deli'.(>I')' 16, 1; FiO" s« o.w.ygen concentration in ,nspired air nuid replaC'm'Ol.'nt therapy 130
gas exchange disorders 18-25 sununary2S g.u exchangE', pulmonary
assessment 52-3 basics 4-17 defLnition 4 glUCO!lt', fa."iting. plasma ~e1s, (OlllU'lOn values for 51
H' ions s« hydrogen tons (W) hilemoglobin (Hb) plasma ronct'nlration 8
rommon values 51 haemoglobin oxygen saturation (SOJ8-11 in arterial bkx>d (~8, SO fal5ely high 81, 120 and V/Q mismatch 14
common values 50 lncrelO5e with tr.msfusion/lron supplements 82-3, 121 HeOl ba\:lnce U'l bicarl:>onate (HCO;J balance
rommon valUl"'li 50 generation 28 in pH ZJ ~~211
hypercapnia 6 aS5eSSment of 5Il?'-"erily 52
chronic t'S ocutt! 7 clinical signs 23 in obesity no in respiratory impilIrmcnt 22 hypen::hlQr.lcmic itd~s 90-1,125 hyperventilation 12, 14,24,25,58-9,
109 in adrenallnsuffidcncy 94-5,127 causes 24 in diabetic kdoacidOSlS 86--7, 123 diagnosis 48 in mesenteric ischaemia 84--5,122
and metabolic acidosis 36 in methanol poisoning 88--9, 124 p;ychogcnic:M, 78--9, 11.'J in renal tubular aodOl>is 90-1. l25 in ~\icylate poOwning 92-3, 126 hyponatf;W'mja~, 127 hypme.aemia 96-7, 118 1ll anaMtia 82-3 asst'$SIr\eJlt of $t'\"erily 52 definition 18 ~ 10 :maemia 127 ~ere80-l, 120
h:.poxia 18, 19 hypm.ic <'Irivc 7, 113 in re;piratory im~innent 22
impaired o:l(}'genatiOCl 18 inadeq\1ale uxygenatiOn 18 Inspired air (inhaled gas) composition during respirahon cycle 13
INDEX eff~lofshunhng
IS
causes 39
oxygen concentraliOll (FK.l.J) 16 ml:et'pretabofl of ABGs 52-5
K- ions 5« potassium (K·) !oRo; kidneys and mo..'tiIbohc alkalosis 38 1\.'ruI1 pH maintenance 28, 29 renallub\llar acidosis, type 1
definition J8 d'illgllOSlS 48 in pylonc steTlosis 130 with vomiting 100-3.130-1 metabolic pH mainte:runce 28, '19 methanol poisoning 88-9, J24 lTllXed i1od
modifioo. Allen lest 44, 47
90--7,125
Kus.sm;I.u1'!> respiration 36
Nil.' ions Ii!« 5Qdium (Nj,°) >OOS nJ~1 prongs 16, 17
ladnte. C(IfJUnOfl values for 51 lactic lIC1dosis 37 III iIoCUle paf\C1'e'antis 98-9,11" In cardiac arrnt 96-7, 128
itl mescnlt>rlc ischaemia
8~';.
122
obtsity 60-1, 1]0 optold lonc:'ly 64-5. 112 olC)'g.:n 10::) Cllncefltrahoo in inspired air (FiOJ 16
mC"enlcric isoehaemia 84--5, III md:,lbolic acidosIS 30, 36--7 in aC\ll~ pilrlCrt"ltilis 98-9. 129
in adrmallll5utficioency 9+-5.717 anicon gap In 39 asst'SSID('flt of .wveritr 54 definition 36 in diabetic keloacidosis 86-7, 123 d,agnosili 4B hyperventilation in 24 in meseoto"!ric is
".
deli>erydeo.·k516-17 effl'd of shunting 15 in pulmonary gas exchange 4 _ also supp&emlmtal O>o:) gen oxygen partial pre5SUIl:' (PO.z) 5 in air 5 In arterial blood (PdoJ 5 iilfId ah-ro1ar ventilation 12-161: common values SO in diagnosis 48 III lmxed aodOSIS 41 in oxyhllemoglobill dissociation 10-11
-'
in respiratn'y unpairmetlt20 common vllues 50 in oxyhaemoglobi.n dis'iOCiiltiOll 10--11 in respiratory impainnenl 25 o")'gemrion 8-11 and ah"eOl.1r vmtilatim> lJ impaired ~~ in..clequale 18
INDEX and oxygen coocentnltion 1f\ inspired air (FiOiI16 oxyhaemoglobin dis5ocialion curve 10-11 in respiratory impairment 20
PlfCO:! s« carbon dIoxide pam",1 pl1'S9ure, in arterial
blood pancreatitis, acute 98-9, J29 P.tOz Sn! oxygen. partial pressure, in artedal blood p.lrtW ~\tres5 sn /il$o carbon dioxide parti.1.1 pressun>; oxygen partial pnossure Pcoz ~ arbon dioxide partial ".,.,."~
pH
blood rommon values 50 in diagnosis 48 IlOrmal26 in respiratory impairment 2J dl!!inition ?J ~le27
_ al$() acid-baSt' balance pickwidian syndrome ItO pneum.orua, aspiration 70-1.115
pOz!# oxygen partial pressure potassium (K') ions in metabolic alkalosis 38 plasm", levels, common values fO!"
51 psychogenic hyperventiiatiOfl 24, 78-9,119 pu1mQnlIry ~lism 62-3, 106-7, 111,13J pulmonary gas exchange us smnent 52-3 bilSics 4--17 definition 4
pulge ox.imetry 8, 59, 109 pylork stCl'l(tiis 102-3, 131
radial artery sampling 44-5 local atllH':5l1wtic 43 wrist position 45 renal mechanisms in metabolic alkalosis 38 pH maintenaru:e 28, 29 renal tubular addaMs, type I 90-7, 125
respiratory acidosis 30, 40 in chronk obslmctive pulmona.f)' ~ 72-5, It4,1J617
chronic us acute 40
COOlpeOS3tedJ2,33 mixed with metabolic acic:b;is
"
in opkrid toxicity 64-5, 112 in I)"e' 2 diabetes 6O--J, no resp;ratory alkalosis 30, 40 in anaemia 82-3,121 in
mixed with metabolic acidosis 7a-I,llS in psychogenic hyperventilation
'"
in pulmonary embolism 62-3, 111 uncompensated 58-9,]09
n:spiralory cen.tIe 6 respiratory compensaltonsn compensation respiratory impairment (RI) diagnosis 48 s«.Iso type J respiratOl)' impairment; type 2 respiratory impairment NSpiratory s)"5lem, pH maintenanre in 28, 29 R.l s« respiratory impairment 139
INDEX
!\alicyl.ale (aspmJ') poi9:lflil'lS 34, 91-3,126 sampl~41 7
asthma 76-7.118 cau.5otS 21 in ctuoruc obstructive pulJJ'\Ol\3.IY d.isl'ase 66-7.]13 -
In
contl'1lirldicarioM 4J puoctun" site!;.u. 4J rolilia1artef}' 43. 44-5 lM-fnnl': samptin,; 0\2-3 "her samplinR.Mj tedmique
markers 21 III pllhnnnary embobsm 62-3. III
equiplJU.'1I1 42
local m"l<--.,;;thPti<' 4.1 1'I......... ni
k~pil'lg 42
wrist poo;ition 45 vcnou.<; \'5 arterial bl\w 44 s.hunting 14, 15 o;odium (Na·) ions ~n metabolic .....i J06is 94-5. 127 11\
rnetaoolJc ;Uk.l\oo<~ 38
plasma ~'C15,. common value;; foe 51 ~truM'
1II.lwte pancreatitis ~8-9, 129 III aspiration pncumwu., 70-1. 115 ~...-.~mmt of sevcnly 20. 21
70-1, 115
:,uppWmenlal oxy~ n j and AUG anaI~ Ib (aution with 73. 75. 117 in respiJatory impairment 20, 22 type 1 R159, 67. 69, 109. 113, 114
lrPatmmt 20 typ'" 2 respiratory in'p.-ljrnloPnt21. 25 olcule-on-d\WlUC 74-.';. 117 CRUseS 23
chrorlll: 25
2.1, 40 III chronic obstructive pUhllQ1li\ry disease 68 9, 72...."0. 114, n6-17 in opioid lQx)oty 64--,.';,112 l.IS ,\(111('
in pylon<: ,.h....1I06i.<; 1JO in type 2 Jiabt-If'"i 60-1. JJO WIth n.-niling 100-3. JJO....1
"r.,
t r('31 mMlt guidance!lJIl,mi toring.,
ABC m49 lyPf! 1 respiratory impainnwt 20-1, 25.~9, 109
140
blood sampk."S 41 ill3dvertmt 104-"0, 1.12 ventilationl p.·rf usion (V IlJ) mismatch 14 illn::.piriltory impaimu.·(,t10 votluting 100-3, 13U-----J and .>cid-base bul.llKX':lR in pyloric 5t~lS 1011-1. 131
Vf'1lOUS
•
EJqlloIno"'" ""rand how ......... OIl ABO DoIoiIs .. tho '.............'0 01 abnanoaI;ty Includrts Q unique ftow.chott for iflll p'elng resuIb 25 purpclt dtnicol em. scenorlos put the ABG
'Nti"'"'
into pradice,
ThIs pocket guide IS flS8f'tticJ reading for 011 tho~ who require greater confidence in performing or interpreting
ABGs and is ideally suiled for medicol students, junior doctors, specialist nurses ond
URCHILL L1V
STONij
_N'"
ww....ebe1'ierbes
nUrH
proclilioners.